PsychiatryBlock 2A

You might also like

Download as pdf or txt
Download as pdf or txt
You are on page 1of 97


1
2 i Item: 1 of 44 ......
=·--e·.,. .,=:J:~-
C>
Next
il
Lab Values
,.
Notes
~
Calculator
• 3 A 45-year-old Caucasian woman comes to the emergency department complaining of a recent onset of anxiety and depression. The
• 4 patient says that she worries about everything and has felt very depressed lately. She complains of a lack of energy, difficulty
• 5 sleeping, and a decreased appetite. She reports difficulties in her relationship with her husband and that she depends on him
• 6 financially. Her husband does not talk to her very much and often leaves home for long periods. She has 6-year-old twin sons who
• 7 were recently diagnosed with autism and require a lot of attention, and she is feeling overwhelmed with her family and financial
• 8 problems. She appears to be an adequate historian. is cooperative during the interview. but appears slightly disheveled. Her mood and
• 9 affect are depressed and anxious. She becomes tearful during the interview. Her now of thought is coherent, and her thought content
• 10 reveals feelings of low self-esteem. There is absence of delusions, or hallucinations. The patient denies suicidal ideas. The patient also
• 1i states that her husband is not violent, and she denies any threats or physical injuries innicted by him. She is oriented to date, place,
• 12 and person with intact recent and remote memory. The patient denies any previous medical or psychiatric problems. Her vital signs
• 13 are normal, and physical examination is unremarkable.
• 14
• 15
Which of the following is the most appropriate next step in management?
• 16
• 17
0 A . Admit the patient to the hospital because of severe depression and anxiety
• 18
• 19 0 B. Call for a psychiatric consult to evaluate for possible depression
• 20
0 C. Call for a social worker consult to assist with management
• 21
• 22 0 D. Discharge patient from the emergency department with antidepressant medication
• 23 0 E. Discharge patient from the emergency department with anxiolytic medication
• 24
• 25 (I 0 F. Discharge patient from the emergency department with outpatient mental health followup

• 26 0 G. Report spousal abuse to social services and inform the patient of available help
• 27
• 28
• 29
• 30 Exit


1
2 i Item: 1 of 44 ......
=·--e·.,. . ,=:J:~-
C>
Next
il
Lab Values
,.
Notes
~
Calculator
• 3
• 4
Explanation Rel<aps & Refs f} Help Q My Questions
• 5
• 6
The correct answer is F. The patient, in this case, clearly has depression and anxiety, which 85% of students got this correct
• 7
most likely relates to a difficult family situation. Because the patient does not express suicidal
• 8 ideation, admission to the hospital is not necessary at this point (c hoice A) . The most important Send us your feedback at:
medFeedback@kaplan.com
• 9 next step in management is to arrange mental health followup in the next couple of days. The
• 10 patient will need to be assessed to determine which treatment, if any, would be most beneficial. Please include QID MB000092

• 1i
There is no need to call for a psychiatric consult in this case (choice B) because there is no
• 12
homicidal or suicidal ideation and no indications for hospital admission.
• 13
• 14 This patient has a complicated family situation, and a social services consult (choice C) may
• 15 be helpful in this case. However, this patienrs primary problem is depression and anxiety, and
• 16 the most appropriate next step in management is to address her concerns and offer appropriate
• 17 mental health assistance. Help from social services will follow in the long-term management plan.
• 18
Prescription of antidepressant (choice D) or anxiolytic (choice E) medications may be
• 19
necessary because this patient presents with a combination of depression and anxiety.
• 20 However, this is not the most appropriate next step in management because a full evaluation has
• 21 not been performed.
• 22
• 23 The patient is experiencing difficulties in her relationship with her husband. Based on the
available information, there is no evidence of spousal abuse (choice G) to report to social
• 24
services.
• 25
• 26 Reviewed on 12/10112
• 27
• 28
• 29
• 30 Exit
• n1c woman whO has no past psycn
• 4 symptoms began 3 months ago when she discovered that her husband was having an affair. Her husband apologized to her and
• 5 promised to end the affair. Despite this, the woman reports feeling betrayed and has feelings of being a bad wife and mother to her
• 6 three children. She also reports that she has been more reluctant to see friends and do her usual hobbies in the past few months and
• 7 wonders if her life is worth living like this. What is the most appropriate next step in the management of this patient?
• 8
• 9
10 0 A. Admit her to the inpatient unit

• 1i 0 B. Arrange for couples therapy
• 12
0 C. Arrange for individual therapy
• 13
• 14 (j 0 0 . Assess for su1c1dal thoughts
• 15
0 E. Prescribe antidepressant medication
• 16
• 17
• 18
• 19
Explanation ReKaps & Refs fi Help Q My Questions

• 20
The correct answer is D. This patient is suffe ring from a major depressive episode in the 81% of students got this correct.
• 21
context of finding out that her husband is having an affair. She requires treatment for her
• 22 Send us your feedback at:
depression. A thorough mental status examination. however. including the assessment for medFeedback@kaplan.com
• 23 safety, needs to be done before deciding the most appropriate treatment plan.
• 24 Please include QID m001048
If this woman had active suicidal ideation, it would be appropriate to admit her to the inpatient
• 25
psychiatric unit (choice A) for more intense monitoring and treatment. We do not know.
• 26
however, that she has active suicidal ideation from the vignette.
• 27
• 28
• 29
• 30 Exit
a Item: 2 of 44 <J C> il , . ~
Prev1ous Next Lab Values Notes Calculator

• 4
0 E. Prescribe antidepressant medication
• 5
• 6
• 7
• 8
Explanation ReKaps & Refs f) Help 0 My Questions

• 9
The c orrect answer i s D. This patient is suffering from a major depressive episode in the 81% of students got this correcL
• 10
context of finding out that her husband is having an affair. She requires treatment for her
• 1i Send us your feedback at:
depression. A thorough mental status examination, however, including the assessment for medFeedback@kaplan.com
• 12
safety, needs to be done before deciding the most appropriate treatment plan.
• 13 Please include QID m001048
• 14 If this woman had active suicidal ideation, it would be appropriate to admit her to the inpatient
• 15 psychiatric unit (choice A) for more intense monitoring and treatment. We do not know,
• 16 however, that she has active suicidal ideation from the vignette.
• 17 Couples therapy (choice B ) may be a part of her overall treatment plan, but it is not the most
• 18 appropriate treatment at this point in time, given her significant depressive symptoms .
• 19
Individual therapy (choice C) may also be a part of this patient's treatment plan, particularly as
• 20
an adjunct to medication, but as a stand-alone, fi rst-line treatment, it is not the most appropriate
• 21
treatment, given the severity of the depressive symptoms described.
• 22
• 23 Antidepressant medication (choice E) is the treatment of choice, together with psychotherapy,
• 24 for the major depressive disorder. Before medication is prescribed, a careful assessment for
• 25 suicidal ideation must be performed.

• 26
• 27
• 28
• 29
• 30 Exit
• 1
• 2
• 3
being treated with sertraline, 150 mg/d for a major depressive episode. He has no prior psychiatric history. He has been on sertraline
• 4
for 2 months. He complains of intermittent nausea that does not significantly affect his life, but he is otherwise doing well on the
• 5
medication, with improvement in all of his symptoms of depression. He is concerned about taking too much medication and asks your
• 6
opinion. Which of the following is the most appropriate management of this patient's medication?
• 7
• 8
• 9 0 A. Change medication to bupropion secondary to nausea
• 10
0 B. Continue sertraline indefinitely
• 11
• 12 fl 0 C. Contmue sertrallne for 6 more months
• 13
0 0 . Gradual taper of sertraline
• 14
• 15 0 E. Stop sertraline; he is doing well
• 16
• 17
• 18 Explanation ReKaps & Refs G Help Q My Questions
• 19
• 20 The correct answer is C. This man presents in the middle of being treated for his first episode 45% of students got this correct.

• 21 of major depression. Adults who have an uncompl icated major depression and a good response
Send us your feedback at:
• 22 to antidepressant medication should continue on the medication for 6 months after achieving full
medFeedback@kaplan.com
• 23 remission .
Please include QID m001053
• 24 Changing the medication to bupropion {choice A) is not the most appropriate treatment,
• 25 because this man is doing well on sertraline and only has mild nausea that does not interfere
• 26 with his functioning .
• 27
• 28
• 29
• 30 Exit


1
2
a Item: 3 of 44 <J C> il , . ~
Prev1ous Next Lab Values Notes Calculator
• 3
• 4 0 D. Gradual taper of sertraline
• 5
0 E. Stop sertraline; he is doing well
• 6
• 7
• 8
• 9
Explanation ReKaps & Refs @ Help 0 My Questions

• 10
The c orrect answer is C. This man presents in the middle of being treated for his first episode 45% of students got this correcL
• 11
of major depression. Adults who have an uncomplicated major depression and a good response
• 12 Send us your feedback at:
to antidepressant medication should continue on the medication for 6 months after achieving full
• 13 medFeedback@kaplan.com
remission .
• 14 Please include QID mOO 1053
• 15 Changing the medication to bupropion (choice A) is not the most appropriate treatment,
• 16 because this man is doing well on sertraline and only has mild nausea that does not interfere
• 17 with his functioning .
• 18 Continuing the medication indefinitely (choice B) is only recommended for adu~s who have
• 19 had more than two previous episodes of major depression The vignette mentions that this is the
• 20 patient's first episode of depression and thus 6 more months of treatment is appropriate.
• 21
Gradual tapering of sertraline (choice D) after only 2 months of treatment is also premature
• 22
and may lead to relapse of depressive symptoms.
• 23
• 24 Discontinuing the sertraline (choice E) after only 2 months of treatment is premature and may
• 25 lead to relapse. Abrupt discontinuation is not recommended.

• 26
• 27
• 28
• 29
• 30 Exit



1
2
3
i Item• 4of44

A ~o-year-old med1ca1 student 1s arrested tor tnreatenmg a a IS I I I I .. I I I'"' ,.


<J
Previous
C>
Next Lab Values
il , .
Notes
o~
~
Calculator

tried to slit his wrists on the way to the police station. His roommate arrives and states that the patient has been doing "weird things"
• 5 over the past two months and has been "acting as if he had no emotions". The patient is socially withdrawn and has been overheard
• 6 saying that the FBI is gMng him tips on patient care and that "people from the sky" are telling him to do "bad things." He says he does
• 7 not want to be a doctor anymore. His face is expressionless and unresponsive. His physical examination is otherwise normal.
• 8 Toxicologic screening is negative. Which of the following is the most likely diagnosis?
• 9
• 10
0 A. Bipolar I disorder
• 11
• 12 0 B. Brief psychotic disorder
• 13
0 C. Delusional disorder
• 14
• 15 0 D. Schizophrenia
• 16 (I 0 E. Schtzophremform disorder
• 17
• 18
• 19
Explanation Rel<aps & Refs 8 Help 0 My Questions
• 20
• 21 86% of students got this correct.
The correct answer is E. Schizophreniform disorder presents with the same symptoms as
• 22
schizophrenia; however, in schizophreniform disorder the symptoms are present for more than
• 23 1 month but less than 6 months. This patient hears voices; has delusional, irrational thoughts;
Send us your feedback at:
medFeedback@kaplan.com
• 24 has a flattened affect and lack of motivation; and has become socially withdrawn. Antipsychotic
• 25 Please include QIO Q0744m
medication is indicated for a course of 3 to 6-months.
• 26
Bipolar I disorder (choice A) is diagnosed after at least one manic episode. It is often
• 27 ...h .......................... ....................................~ ...... ,.. ..... : .......... : ........... . ; .... ....... ;..... ,..,.1,.._.... ,..J" ,.i,.. .......,...,... .... :,...... ..~ ... .... : ................... ,., ........ - · .....

• 28
• 29
• 30 Exit
• 1
• 2
• 3
The correct answer is E. Schizophreniform disorder presents with the same symptoms as 86% of students got this correct.
schizophrenia; however, in schizophreniform disorder the symptoms are present for more than
• 5 Send us your feedback a t:
1 month but less than 6 months. This patient hears voices; has delusional, irrational thoughts;
• 6 medFeedback@kaplan.com
has a flattened affect and lack of motivation; and has become socially withdrawn. Antipsychotic
• 7 Please include QID Q0744m
medication is indicated for a course of 3 to 6-months.
• 8
• 9 Bipolar I disorder (choice A) is diagnosed after at least one manic episode. It is often
• 10 characterized by episodes of mania mixed with episodes of depression. Manic episodes must
be a distinct period of elevated or irritable mood with grandiosity, pressured speech, flight of
• 11
12 ideas, rac ing thoughts, a decreased need for sleep, distractibility, increased goal-directed

behavior, involvement in pleasurable activities with known undesirable consequences (excessive
• 13
shopping, risky sexual activity, unwise business investments). The episodes cause functional
• 14
impairment and last for at least 1 week. Treatment for bipolar disorder is lithium.
• 15
• 16 Brief psychotic disorder (choice B) is characterized by psychotic symptoms that last for more
• 17 than 1 day but less than 1 month. It may be preceded by a stressor, and is self~limited .
• 18 Delusional disorder (choice C) is characterized by nonbizarre delusions (such as being
• 19 poisoned, being followed, or having a disease) that are present for at least 1 month. Treatment
• 20 is psychotherapy.
• 21
Schizophrenia (choice D) is a disease that is characterized by psychotic symptoms (positive
• 22
symptoms), flattening of affect and motivation (negative symptoms), and an impairment of social
• 23
or occupational functioning for at least 6 months. The symptoms are not caused by a mood
• 24
disorder or schizoaffective disorder; or by a medical, neurologic, or substance-induced
• 25
disorder. Treatment consists of antipsychotic agents and psychosocial treatment, including
• 26 reality-based therapy, patient and family support, skill training, and assistance with daily living
• 27 skills.
• 28
• 29
• 30 Exit
• 1
• 2
• 3
A 46-year-old African American man who has a history of chronic schizoaffective disorder is admitted for acute psychosis. He
• 4 presents with auditory hallucinations, paranoia, depression with suicidal ideation, anhedonia, blunted affect, increased appetite, a
• 5 10-lb weight gain over the past month, and increasing somnolence. The patient is stabilized in the hospital with haloperidol. Which of
• 6 the following is the best atypical antipsychotic medication to prescribe for this patient?
• 7
• 8
• 9 0 A. Clozapine
• 10
0 B. Olanzapine
• 11
• 12 0 C. Quetiapine
• 13 0 D. Risperidone
• 14
• 15 (j 0 E. Z1pras1done
• 16
• 17
• 18 Explanation ReKaps & Refs 0 Help Q My Questions

• 19
20 The correct answer is E. This patient presents with classic signs and symptoms of an 36% of students got this correct

21 exacerbation of schizoaffective disorder. The patient has been stabilized with haloperidol, and
• Send us your feedback at:
atypical antipsychotic drugs have been shown to be effective for positive symptoms such as
• 22 medFeedback@kaplan. com
hallucinations and paranoia, but also effective for negative symptoms such as anhedonia and
• 23 Please include QID m000805
blunted affect. The patient also has symptoms of depression. Consequently the patient requires
• 24
an atypical antipsychotic that has activating effects and less sedating effects. Ziprasidone
• 25
causes mild sedation and together with aripiprazole (which is not one of the answer choices). is
• 26 activating initially, and is weight-neutral, an additional advantage for this patient who has recently
• 27 oained 10 lb. Ziorasidone is indicated for the treatment of schizoohrenia. acute aoitation in
• 28
• 29
• 30 Exit


1
2
a Item: 5 of 44 <J C> il , . ~
Prev1ous Next Lab Values Notes Calculator
• 3
• 4
• 5
• 6 Explanation ReKaps & Refs G Help Q My Questions
• 7
• 8 The co rrect answer is E. This patient presents with classic signs and symptoms of an 36% of students got this correct
• 9 exacerbation of schizoaffective disorder. The patient has been stabilized with haloperidol, and
Send us your feedback at:
• 10 atypical antipsychotic drugs have been shown to be effective for positive symptoms such as
medFeedback@kaplan.com
• 11 hallucinations and paranoia, but also effective for negative symptoms such as anhedonia and
blunted affect The patient also has symptoms of depression. ConsequenUy the patient requires Please include QID mOOOBOS
• 12
• 13 an atypical antipsychotic that has activating effects and less sedating effects. Ziprasidone
• 14 causes mild sedation and together with aripiprazole (which is not one of the answer choices), is
• 15 activating initially. and is weight-neutral, an additional advantage for this patient who has recently
• 16 gained 10 lb. Ziprasidone is indicated for the treatment of schizophrenia, acute agitation in
• 17 patients who have schizophrenia, and the treatment of acute manic or mixed episodes
associated with bipolar disorder with or without psychosis
• 18
• 19 Clozapine (choice A) has been noted to cause sedation and has been shown to cause
• 20 substantial weight gain, diabetes, and hypercholesterolemia
• 21
Olanzapine (choice B) and quetiapine {choice C) both cause weight gain and sedation.
• 22
• 23 Risperidone (choice D) causes weight gain. These atypical antipsychotic medications could
• 24 treat the patient's positive and negative symptoms, but these side effects of sedation and weight
• 25 gain would not be beneficial for this patient

• 26
• 27
• 28
• 29
• 30 Exit
• 1
• 2
• 3 A 59-year-old woman had a diverticulectomy. During the observation period, the intern notices that the patient's vital signs are all
• 4 increased and she seems confused, disoriented, and delirious. She starts having seizures. On review of her chart, the intern notices
• 5 that the woman has a past history of hypertension, diabetes mellitus, anxiety, gout, and peMc inflammatory disease. She is on
thiazide, metformin, alprazolam, and allopurinol. Her father has bipolar disorder and is on lithium. She is hepatitis B positive. She is a
• 7 smoker, and drinks only socially. Which of the following would be the most appropriate next step?
• 8
• 9
• 10 0 A. Dialysis to wash out lithium
• 11 0 B. Start dantrolene
• 12
• 13 f) 0 C. Start lorazepam
• 14 0 0 . Start naloxone
• 15
0 E. Start promethazine
• 16
• 17
• 18
• 19
Explanation ReKaps & Refs tj, Help 0 My Questions

• 20
The correct answer is C. The patient takes alprazolam, and is most likely experiencing 65% of students got this correct.
• 21
benzodiazepine withdrawal symptoms. Commonly observed symptoms of benzodiazepine
• 22 Send us your feedback at:
withdrawal include: anxiety, diaphoresis, irritability, insomnia, fatigue , headache, myalgias, medFeedback@kaplan .com
• 23
nausea, perceptual disturbances, tremors, and seizures. The most appropriate management
• 24 Please include QIO Q0079m
step is parenteral administration of a short-acting benzodiazepine, such as lorazepam.
• 25
• 26 Dialysis (c hoice A) could have been the answer if this patient were on lithium. But the stem
• 27 states that it is her father who takes lithium.
• 28
• 29
• 30 Exit


1
2
a Item: 6 of 44 <J C> il , . ~
Prev1ous Next Lab Values Notes Calculator
• 3
• 4 fl 0 C. Start lorazepam
• 5 0 D. Start naloxone

7 0 E. Start promethazine

• 8
• 9
• 10 Explanation ReKaps & Refs 0 Help Q My Questions

• 11
• 12 The correct answer is C. The patient takes alprazolam, and is most likely experiencing 65% of students got this correct.
benzodiazepine withdrawal symptoms. Commonly observed symptoms of benzodiazepine
• 13 Send us your feedback at:
withdrawal include: anxiety, diaphoresis, irritability, insomnia, fatigue, headache, myalgias,
• 14 medFeedback@kaplan.com
nausea, perceptual disturbances, tremors, and seizures. The most appropriate management
• 15 Please include QID Q0079m
step is parenteral administration of a short-acting benzodiazepine, such as lorazepam.
• 16
• 17 Dialysis (choice A} could have been the answer if this patient were on lithium. But the stem
• 18 states that it is her father who takes lithium.
• 19 Dantrolene (choice B) is used in the treatment of neuroleptic malignant syndrome, which is a
• 20 life-threatening adverse effect of antipsychotics characterized by muscle rigidity, hyperthermia,
• 21 autonomic instability, and delirium. It has no role in benzodiazepine withdrawal.
• 22
Naloxone (choice D) is used in suspected morphine intoxication.
• 23
• 24 Promethazine (choice E) is a first generation H 1 receptor antagonist antihistamine and
• 25 antiemetic medication, with strong anticholinergic and sedative effects.
• 26
• 27
• 28
• 29
• 30 Exit
• 1
• 2
• 3
4 A 3-year-old boy is brought to the office by his parents for a regular health supervision examination. His condition has not changed

significantly since his last visit 6 months ago. He still prefers to play by himself. The mother has finally adjusted to the fact that her son
• 5
never returns the hug when she hugs him. On physical examination, the boy is non-cooperative and utters unintelligible mumbles when
• 6
approached him. The physician fails to establish any eye contact with him. The child has had three attacks of otitis media since birth.
• 7
Which of the following is the most likely diagnosis?
• 8
• 9
• 10 0 A. Asperger disorder
• 11
• 12
(I 0 B. Aut1sm
• 13 0 C. Hearing loss due to repeated otitis media
• 14
0 D. Mental retardation
• 15
• 16 0 E. Rett syndrome
• 17
• 18
• 19 Explanation Rel<aps & Refs 0 Help 0 My Questions
• 20
• 21 The correct answer is B . This patient has the typical clinical presentation of autism. Autism is 48% of students got this correct.

• 22 a developmental disorder characterized by impaired social relatedness, deficits in verbal and


Send us your feedback at:
• 23 nonverbal communication, and unusual responses to the environment. II develops before 30 medFeedback@kaplan.com
• 24 months of age and is more common in boys (4:1). The cause is unknown. Clinical features
Please include QIO s2s30 lm
• 25 include failure to attach as an infant, delayed or absent social smile, and failure to anticipate
26 interaction with the caretaker. Patients demonstrate a delay in verbal and nonverbal

communication skills. Stereotypical movements and a need for sameness and routines are
• 27 1"'\ ••, ................ -~ - - - - - .................. -................................................. ~& : .... : •• -: ......... L. .... a... .....
... .......................: .... : ... .: ..... /\ ,., : ....-: ..
• 28
• 29
• 30 Exit
• 1
• 2
• 3
The correct answer is B . This patient has the typical clinical presentation of autism. Autism is 48% of students got this correct.
• 4
a developmental disorder characterized by impaired social relatedness, deficits in verbal and
• 5 Send us your feedback at:
nonverbal communication, and unusual responses to the environment. It develops before 30 medFeedback@kaplan.com
• 6
months of age and is more common in boys (4:1). The cause is unknown. Clinical features
• 7
include failure to attach as an infant, delayed or absent social smile, and failure to anticipate
Please include QID s2s301m
• 8
interaction with the caretaker. Patients demonstrate a delay in verbal and nonverbal
• 9
communication skills. Stereotypical movements and a need for sameness and routines are
• 10 characteristic. Outbursts of anger are common, as well as self-injurious behavior. Autistic
• 11 children are content to play alone. They may or may not be mentally retarded. Treatment
• 12 consists of behavioral and educational programs geared to satisfy the indMdual needs of the
• 13 patient. Pharmacotherapy may be used to treat some target behaviors. Prognosis is poor, and a
• 14 very small number of these children will grow up to be marginally self-sufficient. The vast
• 15 majority end up institutionalized. A better prognosis is associated with patients who demonstrate
• 16 functional speech and higher intelligence.
• 17
Asperger disorder (choice A) is a pervasive developmental disorder that differs from autism in
• 18
that the patients are more communicative, appear more socially aware, and do not have
• 19 language impairments found in autism. They do have social impairments, repetitive behaviors,
• 20 and sometime obsessional interests.
• 21
Hearing loss due to repeated otitis media (choice C) is a strong differential because deafness
• 22
can also cause a child to be socially withdrawn (playing alone, being non-cooperative, and lack
• 23
of eye contact) because the child cannot hear. But this child's lack of emotional attachment and
• 24
his unintelligible mumbles direct the diagnosis more toward autism.
• 25
• 26 Mental retardation (choice D) affects 1 to 3% of the population and occurs more commonly in
• 27 boys. The patient must have both impaired functioning and an intelligence quotient lower than
• 28
• 29
• 30 Exit


1
2
a Item: 7 of 44 <J C> il , . ~
Prev1ous Next Lab Values Notes Calculator
• 3
• 4 consists of behavioral and educational programs geared to satisfy the individual needs of the
• 5 patient. Pharmacotherapy may be used to treat some target behaviors. Prognosis is poor, and a
• 6 very small number of these children will grow up to be marginally self-sufficient. The vast
• 7 majority end up institutionalized. A better prognosis is associated with patients who demonstrate
• 8 functional speech and higher intelligence.
• 9 Asperger disorder (choice A) is a pervasive developmental disorder that differs from autism in
• 10 that the patients are more communicative, appear more socially aware, and do not have
• 11 language impairments found in autism. They do have social impairments, repetitive behaviors,
• 12 and sometime obsessional interests.
• 13
Hearing loss due to repeated otitis media (choice C) is a strong differential because deafness
• 14
can also cause a child to be socially withdrawn (playing alone, being non-cooperative, and lack
• 15
of eye contact) because the child cannot hear. But this child's lack of emotional attachment and
• 16
his unintelligible mumbles direct the diagnosis more toward autism.
• 17
• 18 Mental retardation (choice D) affects 1 to 3% of the population and occurs more commonly in
19 boys. The patient must have both impaired functioning and an intelligence quotient lower than

70, with the onset before 18 years of age. Most cases have no obvious cause. Autistic children
• 20
may or may not be mentally retarded .
• 21
• 22 Rett syndrome (choice E) is an X-linked dominant disorder, thus affecting only girls.
• 23 Development is normal until 1 year of age, when language and motor milestones regress and an
• 24 acquired microcephaly is seen. Hand wringing and sighing are characteristic, and these girls
• 25 develop autistic behavior.
• 26
• 27
• 28
• 29
• 30 Exit
• 1
• 2
• 3
A 24-year-old student is brought to the emergency department because of a sudden change in behavior. His friends report that he has
• 4
been acting "weird" and confused and has been talking about "flying above the floor." He laughs without a reason and then bursts into
• 5
tears. He had been at a party the night before. He appears hot and has uncoordinated movements. He has horizontal nystagmus,
• 6 ataxia, and muscular rigidity, and becomes very agitated and combative during the examination. Which of the following agents should
• 7
be administered at this time to treat his agitation?

• 9
• 10 0 A. Alprazolam
• 11
0 B. Chlorpromazine
• 12
• 13 0 C. Flumazenil
• 14 0 D. Haloperidol
• 15
• 16 0 0 E. M1dazolam
• 17 0 F. Na~rexone
• 18
• 19
• 20 Explanation Rel<aps & Refs @ Help 0 My Questions
• 21
• 22 The correct answer is E . PCP intoxication is characterized by belligerence, assau~iveness, 53% of students got this correct.
• 23 psychomotor agitation, nystagmus, hypertension, seizures, hyperacusis, and coma .
• 24 Send us your feedback at :
PCP-induced agitation and psychosis are treated with benzodiazepines (diazepam, midazolam,
medFeedback@kaplan.com
• 25 or lorazepam) or haloperidol. Benzodiazepines are preferred because antipsychotic
Please include QID s2s152m
• 26 medications (such as chlorpromazine and haloperidol [choices B and D] ), can amplify
• 27 hyperthermia, dystonic reactions, anticholinergic effects, and can even lower the seizure
• 28
• 29
• 30 Exit


1
2
a Item: 8 of 44 <J C> il , . ~
Prev1ous Next Lab Values Notes Calculator
• 3
• 4 0 0 E. M1dazolam

• 5 0 F. NaHrexone
• 6
• 7
Explanation ReKaps & Refs 0 Help 0 My Questions
• 9
• 10 The c orrect answer i s E . PCP intoxication is characterized by belligerence, assauHiveness, 53% of students got this correct.

• 11 psychomotor agitation, nystagmus, hypertension, seizures, hyperacusis, and coma .


Send us your feedback at:
• 12 PCP-induced agitation and psychosis are treated with benzodiazepines (diazepam, midazolam,
medFeedback@kaplan.com
• 13 or lorazepam) or haloperidoL Benzodiazepines are preferred because antipsychotic
Please include QID s2s152m
• 14 medications (such as chlorpromazine and haloperidol (choices B and D] ). can amplify
• 15 hyperthermia, dystonic reactions , anticholinergic effects, and can even lower the seizure
16 threshold. Cranberry juice or ascorbic acid may be useful in acidifying the urine and speeding

• 17 up the elimination of the drug.

• 18 Alprazolam (choice A) is a benzodiazepine used in the treatment of anxiety, panic disorder,


• 19 and phobias .
• 20
Flumazenil (choice C) is used to treat benzodiazepine overdose. It is not used in the treatment
• 21
of PCP intoxication.
• 22
• 23 Naltrexone (choice F) is used in opioid detoxification and significantly shortens the
• 24 detoxification time when combined with clonidine. It is a long-acting oral opioid antagonist that is
• 25 also used in helping to maintain abstinence for up to several months.

• 26
• 27
• 28
• 29
• 30 Exit
• 1
• 2
• 3 A 31-year-old woman with schizophrenia is brought to the emergency department by the police because she was found in a park,
• 4 actively hallucinating and talking to herself. She is cooperative but irritable. She denies command hallucinations and is not violent or
• 5 suicidal. She admits that she ran out of her medication several days ago and was unable to get in touch with her case manager. She
• 6 has been living in a personal care home for the past 3 months and was stable on her medication, but says she left the personal care
• 7 home yesterday and never returned .
• 8
• 9
• 10 Which of the following is the most appropriate next step in management?
• 11
• 12 0 A. Admit her to the psychiatry inpatient service for further treabnent
• 13 fj 0 B. Contact her case manager to ensure support and follow-up
• 14
• 15 0 C. Convince her to take a haloperidol decanoate shot and stay "medicated" for a month
• 16 0 0 . Give her a new prescription and discharge her to the personal care home
• 17
0 E. Send her back to the personal care home and let the owner handle her
• 18
• 19
• 20
• 21
Explanation Rel<aps & Refs @, Help 0 My Questions

• 22
The correct answer is B. The most appropriate step is to contact the case manager to verify 42% of students got this correct
• 23
the woman's information and to ensure that she gets the care and medication she needs. She
• 24 Send us your feedback at :
may be medicated in the emergency department and then sent to the personal care home with
• 25 medFeedback@kaplan.com
the case manager or the personal care homeowner after contacting the case manager about the
• 26 Please include QID s2s1S6m
patient's current condition.
• 27
• 28
• 29
• 30 Exit


1
2
a Item: 9 of 44 <J C> il , . ~
Prev1ous Next Lab Values Notes Calculator
• 3
• 4 0 E. Send her back to the personal care home and let the owner handle her
• 5
• 6
• 7 Explanation ReKaps & Refs G Help Q My Questions
• 8
• 9 The c orrect answer i s B. The most appropriate step is to contact the case manager to verify 42% of students got this correct
• 10 the woman's information and to ensure that she gets the care and medication she needs. She
Send us your feedback at:
• 11 may be medicated in the emergency department and then sent to the personal care home with
medFeedback@kaplan.com
• 12 the case manager or the personal care homeowner after contacting the case manager about the
Please include QID s2s156m
• 13 patienrs current condition.
• 14 Admitting her to psychiatry (choice A) is not justified because she has a case manager and a
• 15 place to go, and is no threat to herself or others.
• 16
Giving her haloperidol decanoate (choice C) is not appropriate because she has been on oral
• 17
atypical medication before and has been doing well. She is compliant with medication, and
• 18
giving a decanoate shot would only increase the risk for tardive dyskinesia.
• 19
• 20 Giving a patient a prescription (choice D) while she is actively hallucinating is not a good
• 21 choice. The chances of her actually having it filled at the pharmacy are minimal.
• 22
Sending her back to the personal care home (choice E) without any intervention is not
• 23 appropriate care.
• 24
• 25 Reviewed on 12/10112
• 26
• 27
• 28
• 29
• 30 Exit
• 1
• 2
• 3
An 86-year-old Caucasian woman is brought in by her daughter. The patient has a past medical history of gout, hypothyroidism, and
• 4
hyperlipidemia. Her daughter is concerned about her because she has been forgetting the names of her grandchildren and friends
• 5 and misplacing items, such as her keys and purse. The problem has been worsening over the past 10 years. The patient lives alone, is
• 6 able to take care of her daily needs, such as cooking, cleaning, paying the bills, and daily hygiene, and denies difficulty with attention
• 7 or orientation. She has been progressively falling asleep earlier and awakening earlier, but denies much trouble with sleep. She
• 8 denies suicidal ideation, feelings of guilt, and difficulty with appetite. What is the most likely cause of her forgetfulness?
• 9

• 11 (I 0 A. Age-assocrated cognitive decline


• 12
0 B. Alzheimer disease
• 13
• 14 0 C. Delirium
• 15 0 D. Depression
• 16
17
0 E. Vascular dementia

• 18
• 19
• 20 Explanation Rel<aps & Refs @ Help Q My Questions

• 21
The correct answer is A. As a person ages, cognitive decline occurs gradually ; however, 74% of students got this correct.
• 22
these changes do not drastically interfere with daily functioning. Common features of this
• 23 Send us your feedback at :
decline are forgetting names, misplacing items, and difficulty with complex problem solving. medFeedback@kaplan .com
• 24
Patients' attention, memory, and orientation are not impaired. Cognition is stable. These
• 25 Please include QIO m000807
features are attributed to the normal loss of neurons, receptors, and changes of enzymes and
• 26
neurotransmitters that occur with age.
• 27
• 28
• 29
• 30 Exit


1
2
a Item: 10of44 <J C> il , . ~
Prev1ous Next Lab Values Notes Calculator
• 3
• 4
The correct answer is A. As a person ages, cognitive decline occurs gradually; however, 74% of students got this correct.
• 5
these changes do not drastically interfere with daily functioning. Common features of this
• 6 decline are forgetting names, misplacing items, and difficulty with complex problem soMng.
Send us your feedback a t:
medFeedback@kaplan.com
• 7 Patients' attention, memory, and orientation are not impaired. Cognition is stable. These
• 8 Please include QID m000807
features are attributed to the normal loss of neurons, receptors, and changes of enzymes and
• 9 neurotransmitters that occur with age.

Alzheimer dementia (choice B) has a gradual onset in which cognition is stable and
• 11
awareness is clear. It is more common in women. The patienfs attention and orientation are
• 12
impaired. Memory is impaired, displaying decreased encoding. Somatic complaints are less
• 13
common and patients do not display focal neurologic signs. Patients have a fluctuating mood.
• 14 Thinking is impoverished and patients have word-finding problems. Insight into their own disease
• 15 is lost. Patients rarely have a past medical history of epilepsy or hypertension. A NeuroScan
• 16 shows atrophy.
• 17
Delirium (choice C) has an acute onset in which cognition fluctuates and awareness of the
• 18
surroundings is decreased. The patient's attention is intact, but orientation and memory are
• 19
impaired. Thinking is disorganized, and patients exhibit word-finding problems.
• 20
• 21 Geriatric patients who have depression (choice D) or pseudodementia may also have
• 22 symptoms similar to dementia. The onset of symptoms is rapid, the duration is relatively short,
• 23 and the patient's mood is consistently depressed. Patient memory fluctuates with decreased
• 24 ability for retrieval. Patients often give up on answering in cognitive tests and highlight their

• 25 mistakes.

• 26 Vascular dementia (choice E) is more common in men. The dementia has a step-wise
• 27 progression, and patients tend to have more somatic complaints than do patients who have
• 28
• 29
• 30 Exit


1
2
a Item: 10of44 <J C> il , . ~
Prev1ous Next Lab Values Notes Calculator
• 3
• 4 neurotransmitters that occur with age.
• 5
Alzheimer dementia (c hoice B) has a gradual onset in which cognition is stable and
• 6
awareness is clear. It is more common in women. The patient's attention and orientation are
• 7
impaired. Memory is impaired, displaying decreased encoding. Somatic complaints are less
• 8 common and patients do not display focal neurologic signs. Patients have a fluctuating mood.
• 9 Thinking is impoverished and patients have word-finding problems. Insight into their own disease
is lost. Patients rarely have a past medical history of epilepsy or hypertension. A NeuroScan
• 11 shows atrophy.
• 12
Delirium (choice C) has an acute onset in which cognition fluctuates and awareness of the
• 13
surroundings is decreased. The patient's attention is intact, but orientation and memory are
• 14
impaired. Thinking is disorganized, and patients exhibit word-finding problems.
• 15
• 16 Geriatric patients who have depression (choice D) or pseudodementia may also have
• 17 symptoms similar to dementia. The onset of symptoms is rapid , the duration is relatively short,
• 18 and the patient's mood is consistently depressed. Patient memory fluctuates with decreased
19 ability for retrieval. Patients often give up on answering in cognitive tests and highlight their

20 mistakes.

• 21 Vascular dementia (choice E) is more common in men. The dementia has a step-wise
• 22 progression, and patients tend to have more somatic complaints than do patients who have
• 23 Alzheimer dementia. Patients display focal neurologic signs and commonly have a past medical
• 24 history of seizures and hypertension. Insight into their disease is retained , and a NeuroScan
• 25 shows infarcts.
• 26
• 27
• 28
• 29
• 30 Exit


1
2
a Item: 11 of 44 <J C> il , . ~
Prev1ous Next Lab Values Notes Calculator
• 3
• 4 A 24-year-old Caucasian man is brought to the emergency department. He was picked up by the police after yelling and cursing at
• 5 another driver who he believed cut him off inappropriately on the highway. He has been stopped by the police in the past and has
• 6 been on probation for multiple driving violations. The young man describes a history of aggressive behavior and states, "I tend to
• 7 over-react." He reports a history of physical fights that have resulted in holes in the walls of his home. He also reports that his mother
• 8 told him he once had a seizure when he had a high fever as a child. He works in a factory as a production assistant and has done well

• 9 at this job for the past 3 years. Which of the following is the most appropriate diagnosis for this patient?

• 10
• 1.1.
0 A. Antisocial personality disorder
• 12
• 13 fl 0 B. lntermrttent explosive disorder
• 14 0 C. Oppositional defiant disorder
• 15
• 16 0 D. Psychotic disorder
• 17 0 E. Seizure disorder
• 18
• 19
• 20 Explanation Rel<aps & Refs @ Help 0 My Questions
• 21
• 22 The correct answer is B. This patient's history of episodes of aggressive behavior that result 70% of students got this correct.
• 23 in violence or destruction of property, and particularly that the degree of aggressiveness
• 24 Send us your feedback at:
expressed is grossly out of proportion to the prec ipitating event, is consistent with a diagnosis of
medFeedback@kaplan.com
• 25 intermittent explosive disorder.
Please include QIO m001056
• 26
Antisocial personality disorder (choice A) is a pervasive pattern of violation of societal rules
• 27 ~n~ tho rinhtc: nf nthorc: Thic rnan'c hoha\Jinr ic rnnro l"nncictont \uifh ~n ovnlncft10 hoha.\tinr
• 28
• 29
• 30 Exit


1
2
a Item: 11 of 44 <J C> il , . ~
Prev1ous Next Lab Values Notes Calculator
• 3
• 4
0 E. Seizure disorder
• 5
• 6
• 7
Explanation ReKaps & Refs @ Help 0 My Questions
• 8
• 9
The c orrect answer i s B. This patienrs history of episodes of aggressive behavior that result 70% of students got this correcL
• 10
in violence or destruction of property, and particularly that the degree of aggressiveness
• 1.1. expressed is grossly out of proportion to the precipitating event, is consistent with a diagnosis of Send us your feedback at:
medFeedback@kaplan.com
• 12 intennittent explosive disorder.
• 13 Please include QID m001056
Antisocial personality disorder (choice A) is a pervasive pattern of violation of societal rules
• 14
and the rights of others. This man's behavior is more consistent with an explosive behavior
• 15
pattern rather than chronic unlawfulness, lying, or irresponsibility.
• 16
• 17 Oppositional defiant disorder (choice C) is a chronic pattern of negative, hostile, and defiant
• 18 behavior that causes clinically significant impainnent in functioning. This patient has intermittent
• 19 aggressive outbursts that are more consistent with a diagnosis of intermittent explosive disorder.
• 20
Psychotic disorder (choice D) is not the most appropriate diagnosis, because there are no
• 21
formal psychotic symptoms given in this vignette
• 22
• 23 Although this man has a history of a febrile seizure as a child, there is no evidence from the
• 24 history that he has a seizure disorder (choice E) . It would be helpful to have more neurologic
• 25 history and a full neurologic examination done before making this diagnosis.

• 26
• 27
• 28
• 29
• 30 Exit
• 1
• 2
• 3
A 27-year-old man with schizophrenia, chronic paranoid type, comes to the clinic for a follow-up visit. He is alert, oriented, and well
• 4 related , and his thought process is completely organized. He is not complaining of any auditory hallucinations and does not seem to
• 5 be responding to internal stimuli. He denies any suicidal or homicidal intent, ideation, or plan. He says he has difficulty staying in his
• 6 chair during the interview and constantly feels an urge to move around. His past medical history is significant for pneumonia 1 month
• 7 ago, when he needed hospitalization and his dose of risperidone was changed. Which of the following is the most appropriate next
• 8 step in management?
• 9
• 10
• 11 0 A. Add an antidepressant

0 B. Add haloperidol to control psychotic agitation


• 13
• 14 f) 0 C. Decrease the dosage of risperidone
• 15 0 D. Increase the dose of risperidone because of incomplete treatment response
• 16
0 E. Recommend electroconvulsive therapy for refractory psychosis
• 17
• 18
• 19
• 20
Explanation Rel<aps & Refs 0 Help Q My Questions

• 21
The correct answer is C . This patient's presentation is one of akathisia, a sense of 64% of students got this correct.
• 22
restlessness and psychomotor agitation that patients can experience while taking antipsychotic
• 23 Send us your feedback at:
medication. The atypical antipsychotic medications (olanzapine, quetiapine, ziprasidone, medFeedback@kaplan.com
• 24
risperidone) all confer less of a risk for the development of akathisia. W ith risperidone in
• 25 Please include QID s2s127m
particular, however, higher dosing is associated with an increased risk for the development of
• 26
akathisia. The patient in question most likely had an escalation in risperidone dose while
• 27 hosoitalized. secondarv to aaitation durina a delirium. Maintainina the oatient on this dosaae
• 28
• 29
• 30 Exit


1
2
a Item: 12of44 <J C> il , . ~
Prev1ous Next Lab Values Notes Calculator
• 3
• 4
• 5 Explanation Rel<aps & Refs f} Help Q My Questions

• 6
• 7 The correct answer is C . This patienrs presentation is one of akathisia, a sense of 64% of students got this correct.

• 8 restlessness and psychomotor agitation that patients can experience while taking antipsychotic
Send us your feedback at:
9 medication. The atypical antipsychotic medications (olanzapine, quetiapine, ziprasidone,
• medFeedback@kaplan.com
risperidone) all confer less of a risk for the development of akathisia. With risperidone in
• 10 Please include QID s2sl27m
particular, however, higher dosing is associated with an increased risk for the development of
• 11
akathisia. The patient in question most likely had an escalation in risperidone dose while
hospitalized, secondary to agitation during a delirium. Maintaining the patient on this dosage
• 13
indefinitely, especially given these side effects, is unnecessary.
• 14
• 15 The patient is not depressed; therefore, antidepressant medication (choice A) is not indicated.
• 16 Additionally , some antidepressants, including selective serotonin reuptake inhibitors, can
• 17 produce psychomotor restlessness.
• 18 The patient is not psychotically agitated (choice B) the addition of haloperidol will likely
• 19 exacerbate his akathisia .
• 20
An increase in risperidone dosage (choice D) will likely increase the patient's complaint of
• 21
restlessness and make him less likely to adhere to treatment.
• 22
• 23 There is no history to suggest thatthis patient is "treatment refractory." Additionally, the
• 24 treatment for treatment-refractory (choice E) schizophrenia that does not involVe catatonia is
• 25 not electroconvulsive therapy.
• 26
• 27
• 28
• 29
• 30 Exit
• 1
• 2
• 3
A 20-year-old man is brought to the clinic by his parents, who are concerned that he is an alcoholic. They want him to check himself
• 4
in for treatment. In a discussion about his drinking habits, the young man brags that he can now drink more before he gets drunk.
• 5
Which of the following statements is true about this patient?
• 6
• 7
• 8 0 A. The ability to drink more is related to the patienfs age
• 9
10
0 B. The patient does not have alcoholism

• 11 0 C. The patient will have withdrawal symptoms on cessation of alcohol
12

fl 0 0 . There IS not suffic1ent information to determine if this patient has alcoholism or not
• 13
• 14 0 E. This patient has developed alcohol dependence
• 15
• 16
• 17 Explanation ReKaps & Refs fl Help Q My Questions
• 18
• 19 The correct answer is D . Diagnostic and Statistical Manual of Mental Disorders, Fourth 58% of students got this correct.

• 20 Edition (DSM-IV) criteria are required to make the diagnosis of alcohol dependence. The
Send us your f eedback at:
• 21 diagnosis requires three of the following eight DSM-IV c riteria medFeedback@kapian. com
• 22 • Continued drinking despite physical or psychologic consequences caused or exacerbated Please include QID mOOO 150m
• 23 by alcohol
• 24
25 • Neglect of other activities

• 26 • Inordinate time spent drinking and recovering
• 27
• 28
• 29
• 30 Exit


1
2
a Item: 13 of 44 <J C> il , . ~
Prev1ous Next Lab Values Notes Calculator
• 3
• 4 The correct answer is D . Diagnostic and Statistical Manual of Mental Disorders, Fourth 58% of students got this correct.

• 5 Edition (DSM-IV) criteria are required to make the diagnosis of alcohol dependence. The
Send us your feedback at:
• 6 diagnosis requires three of the following eight DSM-IV criteria: medFeedback@kaplan.com
• 7 • Continued drinking despite physical or psychologic consequences caused or exacerbated Please include QID mOOOlSOm
• 8 by alcohol
• 9
10 • Neglect of other actMties

• 11 • Inordinate time spent drinking and recovering
• 12
• Drinking more or over a longer period than intended
• 13
• 14 • Inability to control drinking
• 15
• Tolerance (defined as increased amounts needed for effect)
• 16
• 17 • Withdrawal symptoms on cessation of alcohol
• 18 • Drinking to relieve or avoid withdrawal symptoms
• 19
The present case describes only one of these eight crite ria, tolerance, which is the
• 20
phenomenon of a drinker needing greater amounts of alcohol to gel the same effect.
• 21
• 22 Tolerance develops over time and is related to the activation of live r enzymes that eliminate
• 23 alcohol from the body faster, not necessarily to the person's age (choice A) Because this
• 24 case does not give complete information about the patient, three criteria needed to make the
• 25 diagnosis of alcohol dependence cannot be found.
• 26 Statements such as ''The patient does not have alcoholism" (choice B) or ''This patient has
• 27 developed alcohol dependence" (choice E) are incorrect because the diaQnosis of
• 28
• 29
• 30 Exit


1
2
a Item: 13 of 44 <J C> il , . ~
Prev1ous Next Lab Values Notes Calculator
• 3
• Neglect of other act1V1t1es
• 4
• 5 • Inordinate time spent drinking and recovering
• 6
• Drinking more or over a longer period than intended
• 7
• 8 • Inability to control drinking
• 9 • Tolerance (defined as increased amounts needed for effect)
• 10
• Withdrawal symptoms on cessation of alcohol
• 11
• 12 • Drinking to relieve or avoid withdrawal symptoms
• 13
The present case describes only one of these eight criteria, tolerance, which is the
• 14
phenomenon of a drinker needing greater amounts of alcohol to get the same effect.
• 15
• 16 Tolerance develops over time and is related to the activation of liver enzymes that eliminate
• 17 alcohol from the body faster, not necessarily to the person's age (choice A) Because this
• 18 case does not give complete information about the patient, three criteria needed to make the

19 diagnosis of alcohol dependence cannot be found.



• 20 Statements such as ''The patient does not have alcoholism" (choice B) or ''This patient has
• 21 developed alcohol dependence" (choice E) are inco rrect because the diagnosis of
• 22 dependence requires three out of eight criteria, but not enough information was given in this
• 23 question.
• 24 Withdrawal symptoms on cessation of alcohol (choice C) represent one of the diagnostic
• 25 criteria of dependence, but is not necessarily related to tolerance.
• 26
• 27
• 28
• 29
• 30 Exit


1
2
a Item: 14 of 44 <J C> il , . ~
Prev1ous Next Lab Values Notes Calculator
• 3
• 4 A 51 -year-old man comes to the emergency department several days after being discharged from the hospital where he had an
• 5 extensive workup for abdominal pain; the workup was negative. He now complains of similar abdominal pain, and when the physician
• 6 tries to reassure him that his physical findings and studies are normal, he refuses to leave. accusing the medical team of "not doing
• 7 enough." He insists on calling the surgeon. Instead, a psychiatry consult is called. The psychiatrists describe the patient's mental
• 8 status as normal. There is no evidence of suicidal or homicidal ideation. The interview with the patient, however, reveals that 6 weeks

• 9 ago, after 24 years of marriage, he lost his wife in a car accident. Which of the following is the most appropriate initial step in

• 10 management?

• 11
• 12
0 A. Admit the patient to psychiatry for further evaluation
• 13
(I 0 B. Explore the patrenrs feelings about the recent loss
• 15 0 C. Order an abdominal x-ray
• 16
• 17 0 D. Prescribe an antidepressant

• 18 0 E. Prescribe benzodiazepines on an as-needed basis


• 19
0 F. Reassure the patient that his pain is most likely stress-related
• 20
• 21
• 22
• 23
Explanation Rel<aps & Refs ti Help 0 My Questions

• 24
The correct answer is B. This patient is obviously having a hard time, struggling with feelings 21 % of students got this correct
• 25
of loss and grief. It is easier and socially more acceptable to have a somatic disease. such as
• 26 Send us your feedback a t:
abdominal pain. The patient should be encouraged to process his feelings of pain and anger
• 27 medFeedback@kaplan.com
~hn11t h ie uJi fo'c Ac~fh

• 28
• 29
• 30 Exit


1
2 i Item: 14 of 44 <J
Previous
C>
Next
il
Lab Values
,.
Notes
~
Calculator


3
4
Explanation Rel<aps & Refs ? . . •
• 5
The correct answer is B. This patient is obviously having a hard time, struggling with feelings 21 % of students got this correct
• 6
of loss and grief. It is easier and socially more acceptable to have a somatic disease, such as
• 7 Send us your feedback a t:
abdominal pain. The patient should be encouraged to process his feelings of pain and anger
• 8 medFeedback@kaplan.com
about his wife's death.
• 9 Please include QID s2sl50m
• 10 Admitting the patient to psychiatry (choi ce A) is not appropriate because there are no criteria
• 11 met for admission. He has not expressed any suicidal or homicidal feelings, and at this time it is
• 12 appropriate to discuss his feelings about the loss of his wife. His condition can most likely be
• 13 treated on an outpatient basis.

Ordering an abdominal x-ray (choice C) is a waste of money. The abdominal pain is probably
• 15 functional and not organic .
• 16
Prescribing an antidepressant (choice D) may be an option after a patient is seen on an
• 17
outpatient basis and if he meets the criteria for depression In the emergency department it
• 18
would not be appropriate, because the patient needs a follow-up and full evaluation first.
• 19
• 20 Prescribing benzodiazepines on an as-needed basis (choice E) may be appropriate on a

• 21 short-term basis only, once the patient's feel ings have been explored.
• 22 Reassurance that the pain is most likely stress-related (choice F) may not be appropriate
• 23 because it has not been effective in the past. In addition, the patient needs to be aware of what
• 24 the feelings are rather than simply accepting an explanation. Processing the grief might help
• 25 relieve the symptoms.
• 26
Reviewed on 12/10112
• 27
• 28
• 29
• 30 Exit
• 1
• 2
• 3 A 52-year-old African American woman complains of outbursts of crying spells, hopelessness, guilt, inability to concentrate, loss of
• 4 appetite, sleep problems, and memory problems. She says she would like to be with her son, who died 5 weeks ago, even though she
• 5 denies suicidal ideation, intent, or plan. She expresses extreme anger at God and questions her religious fa ith. She states that she has
• 6 no interest in activities that used to bring pleasure. Which of the following is the most appropriate next step in the management of this
• 7 patient?
• 8
• 9
• 10 (I 0 A. No treatment necessary
• 11 0 B. Prescribe amitriptyline
• 12
• 13 0 C. Prescribe oxazepam
• 14 0 0 . Prescribe sertraline
• 15
0 E. Psychiatric consultation
• 16
• 17 0 F. Refer patient for psychotherapy
• 18
• 19
• 20 Explanation Rel<aps & Refs G Help Q My Questions
• 21
• 22 The correct answer is A. Bereavement, or normal grief, is often characterized by many of the 56% of students got this correct.
• 23 same characteristics as depression, including sadness, tearfulness, loss of appetite, poor sleep,
Send us your feedback at:
• 24 and diminished interest in activities that used to bring pleasure. Illusions or hallucinations of the
medFeedback@kaplan.com
• 25 deceased person are common. Of importance in differentiating grief from major depressive
Please include QID mOOO 133m
• 26 disorder is that people with grief have symptoms that are time-limited, whereas people with major
• 27 depression cannot imagine ever feeling better. The uncomplicated grief reaction can last several
• 28
• 29
• 30 Exit


1
2
a Item: 15of44 <J C> il , . ~
Prev1ous Next Lab Values Notes Calculator
• 3
• 4
• 5 The correct answer is A. Bereavement, or normal grief, is often characterized by many of the 56% of students got this correct

• 6 same characteristics as depression, including sadness. tearfulness. loss of appetite, poor sleep,
Send us your feedback at:
• 7 and diminished interest in activities that used to bring pleasure. Illusions or hallucinations of the
medFeedback@kaplan.com
• 8 deceased person are common. Of importance in differentiating grief from major depressive
disorder is that people with grief have symptoms that are time-limited, whereas people with major Please include QID m000133m
• 9
depression cannot imagine ever feeling better. The uncomplicated grief reaction can last several
• 10
months or longer, depending on the relationship to the deceased. Grief is not considered
• 11
pathologic unless lasting for >1 year. In this case, the patient does not have any evidence of
• 12
psychosis, eliminating brief psychotic disorder and paranoid schizophrenia from the differential
• 13
diagnosis. Also, this patient has no history to suggest a panic disorder, which is a syndrome that
• 14
consists of discrete periods of intense fear or discomfort accompanied by somatic complaints.
• 15 such as palpitations. trembling, shortness of breath, or sweating. There is no need for special
• 16 treatment in the case of normal grief. Support by family members and friends is the best
• 17 remedy.
• 18
Because this patient does not have depression, there is no need to initiate therapy with
• 19
antidepressants such as amitriptyline {choice B) or sertraline (choice D) . Amitriptyline is a
• 20
tricyclic antidepressant and sertraline is a selective serotonin reuptake inhibitor.
• 21
• 22 Oxazepam (choice C) is an oral benzodiazepine used for the management of symptoms
• 23 associated with anxiety disorders and for anxiety associated with depression.
• 24 Psychiatric consultation (choice E) and referral for psychotherapy (choice F) are not
• 25 necessary in the case of bereavement.
• 26
• 27 Reviewed on 12/10112
• 28
• 29
• 30 Exit
• 1
• 2
• 3
A 9-year-old boy comes to a new physician with his mother because of a significant decline in functioning and increased social
• 4
withdrawal. He has always had difficulties at school and was timid and anxious when speaking in front of others. Looking back, the
• 5
mother remembers that she had first noticed problems when he started at a new school after the family relocated because of her
• 6
husband's new job a year ago. After a month in the new school, she had taken him to a pediatrician because of recurrent episodes of
• 7
abdominal pain, nausea, and tiredness that caused repeated absences from school. All medical evaluations have been negative but
• 8 his symptoms seem to persist. Which of the following is the most likely diagnosis?
• 9
• 10
• 11 0 A. Adjustment disorder with anxiety somatization disorder
• 12
0 B. Fact.itious disorder
• 13
• 14 0 C. Hypochondriasis
• 15 (I 0 D. Separation anxiety disorder

• 17 0 E. Somatization disorder

• 18
• 19
• 20 Explanation Rel<aps & Refs 0 Help Q My Questions

• 21
• 22 The correct answer is 0 . Children with separation anxiety disorder (school phobia) usually 38% of students got this correct.

• 23 develop physical symptoms when they are separated from home or from people they are
Send us your feedback at:
24 attached to. It is developmentally inappropriate, and excessive anxiety evidenced by symptoms
• medFeedback@kaplan.com
present at least 4 weeks. The most common ones include repeated physical complaints,
• 25 Please include QIO s2s166
repeated nightmares, excessive worry about losing significant paras, reluctance to attend school
• 26
because of fear of separation, and fear that an untoward event will lead to separation from a
• 27
• 28
• 29
• 30 Exit
• 1
• 2
• 3
• 4 Explanation Rel<aps & Refs f} Q
Help My Questions
• 5
• 6 The correct answer is D . Children with separation anxiety disorder (school phobia) usually 38% of students got this correct
• 7 develop physical symptoms when they are separated from home or from people they are
• 8 Send us your feedback at:
attached to. It is developmentally inappropriate, and excessive anxiety evidenced by symptoms
medFeedback@kaplan.com
• 9 present at least 4 weeks. The most common ones include repeated physical complaints,
• 10 Please include QID s2sl66
repeated nightmares, excessive worry about losing significant paras, reluctance to attend school
• 11 because of fear of separation, and fear that an untoward event will lead to separation from a
• 12 major attachment para. The symptoms are not caused by a general medical condition or other
• 13 developmental disorder. The onset is before the age of 18 years.
• 14 Adjustment disorder with anxiety (choice A) is a development of emotional or behavioral
• 15 symptoms as a response to a known stressor. The disorder is time-limited, usually beginning
within 3 months of the stressful event, and symptoms lessen within 6 months upon removal of the
• 17 stressor or when new adaptation occurs. According to predominant manifestations they can be
• 18 divided into: depressed, anxious, mixed, or with disturbance of conduct.
• 19
Factitious disorder (choice B ) typically begins in early adult life through discrete episodes of
• 20
treatment seeking, following intentional production of physical or psychologic symptoms. The
• 21
motivation is to assume a sick role in the absence of external incentives for such behavior. The
• 22
symptoms may be predominantly physical, psychologic, or combined.
• 23
• 24 Hypochondriasis (choice C) is a preoccupation with fears of having a serious disease based
• 25 on misinterpretation of bodily symptoms. It persists despite normal medical findings and it

26 causes significant distress in everyday functioning. The disturbance lasts at least 6 months.

• 27 Somatization disorder (choice E) requ ires the development of many physical compla ints
• 28
• 29
• 30 Exit


1
2
a Item: 16of44 <J C> il , . ~
Prev1ous Next Lab Values Notes Calculator
• 3
medFeedback@kaplan .com
• 4 present at least 4 weeks. The most common ones include repeated physical complaints,
repeated nightmares, excessive worry about losing significant paras, reluctance to attend school Please include QID s2s 166
• 5
• 6 because of fear of separation, and fear that an untoward event will lead to separation from a
• 7 major attachment para. The symptoms are not caused by a general medical condition or other
• 8 developmental disorder. The onset is before the age of 18 years.
• 9 Adjustment disorder with anxiety (choice A) is a development of emotional or behavioral
• 10 symptoms as a response to a known stressor. The disorder is time-limited, usually beginning
• 11 within 3 months of the stressful event, and symptoms lessen within 6 months upon removal of the
• 12 stressor or when new adaptation occurs. According to predominant manifestations they can be
• 13 divided into: depressed, anxious, mixed, or with disturbance of conduct.
• 14
Factitious disorder (choice B) typically begins in ear1y adult life through discrete episodes of
• 15
treatment seeking, following intentional production of physical or psychologic symptoms. The
motivation is to assume a sick role in the absence of external incentives for such behavior. The
• 17 symptoms may be predominantly physical, psychologic, or combined.
• 18
Hypochondriasis (choice C) is a preoccupation with fears of having a serious disease based
• 19
on misinterpretation of bodily symptoms It persists despite normal medical findings and it
• 20
causes significant distress in everyday functioning The disturbance lasts at least 6 months.
• 21
• 22 Somatization disorder (choice E) requires the development of many physical complaints
• 23 before the age of 30 and occurs over the course of several years. II causes significant
• 24 impairment in functioning . Four pain symptoms, two gastrointestinal symptoms, one sexual
• 25 symptom, and one pseudoneurologic symptom must be present to make this diagnosis.
• 26
• 27
• 28
• 29
• 30 Exit


1
2
a Item: 17of44 <J C> il , . ~
Prev1ous Next Lab Values Notes Calculator
• 3
• 4 A 35-year-old woman presents to the outpatient clinic for evaluation. She reports that the apartment building where she lives had a
• 5 serious fire approximately 1 year ago, and she narrowly escaped. Many others were injured and had to be taken to the hospital for
• 6 medical care. Since that time, she has had nightmares about being trapped in her apartment and unable to escape, she cannot
• 7 concentrate at work because she often th inks of the fire. and she even feels scared when she sees someone light a match. She has
• 8 not been participating in her usual hobbies and feels detached from her friends and family. She reports difficulty falling asleep,

• 9 because she is afraid she will have more nightmares. as well as reports a poor appetite, and a short fuse with coworkers and friends.

• 10 Which of the following is the most appropriate intervention for this patient?

• 11
• 12
0 A. Hospitalization
• 13
• 14 0 B. Long-term psychodynamic psychotherapy
• 15 0 C. Psychologic debriefing of the trauma
• 16
0 D. Start alprazolam
• 2.7
• 18 f) 0 E. Start sertrallne
• 19
• 20
• 21 Explanation Rel<aps & Refs @ Help 0 My Questions
• 22
• 23 The correct answer is E. This young woman is suffering from post-traumatic stress disorder 45% of students got this correct.
• 24 (PTSD) after narrowly escaping a serious fire in her apartment building. Her recurrent thinking
Send us your feedback at:
• 25 about the event. distressing dreams, decreased interest in participation in social activities. sleep medFeedback@kaplan.com
• 26 difficulties, and irritability are all consistent with this diagnosis. SSRis such as sertraline are
Please include QID m001055
• 27 recommended as first-line medication treatment for individuals who have PTSD and have been
• 28
• 29
• 30 Exit


1
2
a Item: 17of44 <J C> il , . ~
Prev1ous Next Lab Values Notes Calculator
• 3
• 4
• 5 Explanation ReKaps & Refs 6 Help 0 My Questions

• 6
• 7 The correct answer is E. This young woman is suffering from post-traumatic stress disorder 45% of students got this correct.

• 8 (PTSD) after narrowly escaping a serious fire in her apartment building. Her recurrent thinking
Send us your feedback at:
• 9 about the event, distressing dreams, decreased interest in participation in social actMties, sleep medFeedback@kaplan.com
10 difficulties, and irritability are all consistent with this diagnosis. SSRis such as sertraline are
• Please include QID mOOlOSS
recommended as first-line medication treatment for indMduals who have PTSO and have been
• 11
associated with the relief of the core PTSD symptoms.
• 12
• 13 Hospitalization (choice A) is not usually necessary as an initial treatment of PTSO, unless
• 14 indMduals develop serious suicidal or homicidal ideation or become unable to care for
• 15 themselves.
• 16 long-term psychodynamic psychotherapy (choice B) may be useful in addressing
• 2.7 developmental or interpersonal issues that relate to the nature, severity, or symptoms of PTSD,
• 18 but it has not been shown to be an effective stand-alone treatment for PTSO.
• 19
Psychologic debriefing (choice C) is generally not recommended for the treatment of PTSD,
• 20
because it has been shown to increase symptoms in some settings and seems to be ineffective
• 21
in treating individuals who have PTSD.
• 22
• 23 Alprazolam (choice D) may be useful in reducing this woman's anxiety symptoms but would
• 24 not be recommended as monotherapy, because benzodiazepines have not been shown to
• 25 reduce core symptoms of PTSD and clinical observations include the possibility of
• 26 dependence.
• 27
• 28
• 29
• 30 Exit


1
2
a Item: 18of44 <J C> il , . ~
Prev1ous Next Lab Values Notes Calculator
• 3
• 4 A philosophy student has always been different from others and believes in supernatural powers. She thinks that at times she hears
• 5 the voice of God. She believes that there is evil in the form of witchcraft in the world but feels protected because she "can spread an
• 6 energy shield around" her body. She has tried to be a medium for communicating with the deceased based on what she thought was
• 7 one communication with her dead grandmother. Some of her friends think she is a little bit odd, but she feels that is okay because
• 8 nothing is really wrong with her. She admits that she is quite different from the rest of her siblings and acquaintances. She is

• 9 otherwise functional. Which of the following is the most likely diagnosis?

• 10
• 11
0 A. Avoidant personality disorder
• 12
• 13 0 B. Mania
• 14 0 C. Paranoid personality disorder
• 15
16 0 D. Schizoid personality disorder

• 17 (I 0 E. Sch1zolypal personality disorder

• 19
• 20 Explanation Rel<aps & Refs 6 Help Q My Questions
• 21 '

• 22 The correct answer is E . Schizotypal personality disorder is characterized by a pervasive 41 % of students got this correct.
• 23 pattern of social and interpersonal deficits and eccentric behavior as indicated by five or more
Send us your feedback at:
• 24 of the following characteristics: medFeedback@kaplan.com
• 25
• Odd beliefs Please include QID s2s175m
• 26
• Unusual perceptual experiences
• 27 • I l'lr.k nf r.ln<:A friAnt1<: c:uc:nir.inu<:nA<:<:
• 28
• 29
• 30 Exit
• 1
• 2
• 3 Rel<aps & Refs
• 4
• 5 The correct answer is E . Schizotypal personality disorder is characterized by a pervasive 41 % of students got this correct
• 6 pattern of social and interpersonal deficits and eccentric behavior as indicated by fiVe or more
• 7 Send us your feedback at:
of the following characteristics: medFeedback@kaplan.com
• 8
• Odd beliefs Please include QID s2sl75m
• 9
• Unusual perceptual experiences
• 10
• Lack of close friends, suspiciousness
• 11
• Odd speech
• 12
• Inappropriate affect
• 13
• Odd appearance
• 14
• 15 Avoidant personality disorder (choice A) is characterized by a pervasive pattern of social
inhibition, feelings of inadequacy, and hypersensitivity to negative evaluation, seen as: inhibition
• 16
in interpersonal situations, a view of oneself as socially inept, restraint in intimate relationships ,
• 17
an unwillingness to get involved with people unless certain of being liked, and the fear of being
rejected in social situations.
• 19
• 20 Mania (choice B) is characterized by a decreased need for sleep, pressured speech,
• 21 increased libido, reckless behavior without regard fo r consequences, grandiosity (e.g .. special
• 22 powers from God), and severe thought disturbances, which may or may not include psychosis.
• 23 Even though this patient believes in supernatural powers and can hear God's voice, she does
• 24 not have a sense of grandiosity.

• 25 Paranoid personality disorder (choice C) is characterized by a pervasive distrust and


• 26 suspic iousness of others in a way that their actions are interpreted as malevolent, as indicated
• 27 by: reluctance to confide in others, preoccupation with unjustified doubts, suspicions that
• 28
• 29
• 30 Exit


1
2
a Item: 18of44 <J C> il , . ~
Prev1ous Next Lab Values Notes Calculator
• 3
• Odd speech
• 4
• Inappropriate affect
• 5
• Odd appearance
• 6
• 7 Avoidant personality disorder (choice A) is characterized by a pervasive pattern of social
• 8 inhibition, feelings of inadequacy, and hypersensitivity to negative evaluation, seen as: inhibition
in interpersonal situations, a view of oneself as socially inept, restraint in intimate relationships,
• 9
an unwillingness to get involved with people unless certain of being liked, and the fear of being
• 10
rejected in social situations.
• 11
• 12 Mania (choice B) is characterized by a decreased need for sleep, pressured speech,
• 13 increased libido, reckless behavior without regard for consequences, grandiosity (e.g ., special
• 14 powers from God), and severe thought disturbances, which may or may not include psychosis.
• 15 Even though this patient believes in supernatural powers and can hear God's voice, she does
• 16 not have a sense of grandiosity.
• 17 Paranoid personality disorder (choice C) is characterized by a pervasive distrust and
suspiciousness of others in a way that their actions are interpreted as malevolent, as indicated
• 19 by: reluctance to confide in others, preoccupation with unjustified doubts, suspicions that
• 20 others are being exploitative, and finding hidden meaning in benign remarks.
• 21
Schizoid personality disorder (choice D) is characterized by the pervasive pattern of
• 22
detachment from social relationships and restricted emotions, as evidenced by four or more of
• 23 the following characteristics: lack of desire for close relationships, no interest in sexual
• 24 experiences, indifference to praise or criticism, emotional coldness, and choosing solitary
• 25 activities.
• 26
• 27
• 28
• 29
• 30 Exit
• 1
• 2
• 3
4 A 69-year-old woman is brought to her primary care physician for her routine yearly examination. This year, her daughter drove her

to the appointment. When asked about this, the daughter whispers to the physic ian that her mother has gotten lost while driving to the
• 5
supermarket on two occasions over the past 6 months. It becomes clear during the history that the woman does not remember the
• 6
physic ian's name and seems to have some word-finding difficulties. The daughter also mentions to the physician that she has recently
• 7
begun managing her mother's checking account and other finances, due to the mother's forgetfulness. The patient has osteoarthritis
• 8
of the knees, and a history of "a treated, very serious sexually transmitted disease many years ago," but is otherwise healthy. Vital
• 9 signs are within normal limits, and physical examination is unremarkable. An informal mental status examination reveals slow thinking
• 10 and impaired attention. Which of the following is the most appropriate next step for evaluating this patienrs memory decline?
• 11
• 12
• 13 0 A. Computed tomography (CT) scan of the brain
• 14
0 B. ELISA for HIV
• 15
• 16 0 C. Lumbar puncture
• 17 (I 0 D. M1n1-Mental State Examination
• 18
0 E. Referral for neuropsychological testing
• 19
• 20 0 F. RPR (Rapid Plasma Reagin)
• 21
• 22
• 23 Explanation ReKaps & Refs G Help Q My Questions
• 24
• 25 The correct answer is D. The Mini-Mental State Examination (MMSE) is a brief instrument 38% of students got this correct.
• 26 designed to grossly assess cognitive functioning. It assesses orientation, memory, calculation,
27 Send us your feedback at:
• read ing and writing capac ity, visuospatial ability, and language. The maximum score is 30. Th is fTicl'lt=ccrlh~rLo

• 28
• 29
• 30 Exit


1
2
a Item: 19of44 <J C> il , . ~
Prev1ous Next Lab Values Notes Calculator
• 3
• 4 The correct answer is D. The Mini-Mental State Examination (MMSE) is a brief instrument 38% of students got this correct.
• 5 designed to grossly assess cognitive functioning. It assesses orientation, memory, calculation,
• 6 Send us your feedback a t:
reading and writing capacity, visuospatial ability, and language. The maximum score is 30. This
medFeedback@kaplan.com
• 7 tool is widely used as a simple, quick assessment of possible cognitive deficits. The MMSE can
• 8 give the clinician a crude measurement of cognitive decline, and the tool can be re-ad ministered Please include QID Q0299m

• 9 periodically to follow progression of dementia.


• 10
Neuroimaging studies, such as a CT scan {choi ce A) , may be obtained as part of the workup
• 11
for dementia, especially to rule out reversible causes of dementia such as intracranial mass
• 12
lesions, recurrent cerebrovascular accidents, and normal pressure hydrocephalus. But aCT
• 13 should definitely not be the first step. We need to diagnose and assess the degree of dementia,
• 14 using the MMSE first. Then we can look for the cause.
• 15
This patient could have HIV dementia, as indicated by her having serious STD in the past, for
• 16
which an ELISA (choice B) may be necessary later on. But we need to diagnose and assess
• 17
the degree of dementia first, before looking for the cause.
• 18
• 19 A lumbar puncture (choice C) may be indicated if serum syphilis testing is reactive or in the
• 20 evaluation of normal pressure hydrocephalus.
• 21 Neuropsychological testing (choice E) may be benefic ial in differentiating dementia from
• 22 other neuropsychiatric syndromes, for assessing the degree of functional impairment, and for
• 23 assessing specific areas of significant cognitive decline.
• 24
RPR (c hoice F) may be necessary later. But we need to diagnose and assess the degree of
• 25
dementia first, before looking for the cause.
• 26
• 27 Some of the important laboratory studies in the workup of dementia include basic chemistries;
• 28
• 29
• 30 Exit


1
2
a Item: 19of44 <J C> il , . ~
Prev1ous Next Lab Values Notes Calculator
• 3
• 4 Neuroimaging studies, such as a CT scan (choice A) , may be obtained as part of the workup
• 5 for dementia, especially to rule out reversible causes of dementia such as intracranial mass
• 6 lesions, recurrent cerebrovascular accidents, and normal pressure hydrocephalus. But a CT
• 7 should definitely not be the first step. We need to diagnose and assess the degree of dementia,
• 8 using the MMSE first Then we can look for the cause .
• 9
This patient could have HIV dementia, as indicated by her having serious STD in the past, for
• 10
which an ELISA (choice B) may be necessary later on. But we need to diagnose and assess
• 11
the degree of dementia first, before looking for the cause.
• 12
• 13 A lumbar puncture (choice C) may be indicated if serum syphilis testing is reactive or in the
• 14 evaluation of normal pressure hydrocephalus.
• 15 Neuropsychological testing (choice E) may be beneficial in differentiating dementia from
• 16 other neuropsychiatric syndromes, for assessing the degree of functional impairment, and for
• 17 assessing specific areas of significant cognitive decline .
• 18
RPR (choice F) may be necessary later. But we need to diagnose and assess the degree of
• 19
dementia first, before looking for the cause.
• 20
• 21 Some of the important laboratory studies in the workup of dementia include basic chemistries;
• 22 complete blood counts; liver, renal, and thyroid function studies; syphilis serology; serum folate
• 23 and vitamin B12 levels; erythrocyte sedimentation rate; and HIV screening. Other studies may

• 24 be included as indicated by history and physical examination.

• 25 Reviewed on 712112
• 26
• 27
• 28
• 29
• 30 Exit


1
2
a Item: 20 of 44 <J C> il , . ~
Prev1ous Next Lab Values Notes Calculator
• 3
• 4 A 19-year-old woman is brought to the emergency department by her friends after a week-long history of "bizarre behavior." The
• 5 friends say that she has been ''talking a mile a minute" and stating that she has special powers from God. The patient has not slept in
• 6 the past 3 days and yet has been "running all over the place and spending to the limit on her credit cards." On mental status
• 7 examination, the patient is distracted with pressured speech and flight of ideas. When asked about her mood, the patient replies that
• 8 she has never felt this good before. She denies any suicidal or homicidal intent, ideation, or plan. She has never been depressed in

• 9 her life. There is no history of substance abuse, and urinary toxicology is negative. Which of the following is the most likely diagnosis?

• 10
• 11 fl 0 A. B1polar disorder, type I
• 12
• 13 0 B. Bipolar disorder, type II
• 14 0 C. Cyclothymic disorder
• 15
0 D. Major depressive disorder with atypical features
• 16
• 17 0 E. Mood disorder not otherwise specified (NOS)
• 18
0 F. Schizophrenia, chronic paranoid type
• 19

• 21
• 22
Explanation ReKaps & Refs 0 Help Q My Questions

• 23
The correct answer is A . This patient is most likely experiencing a manic episode that does 84% of students got this correct.
• 24
not appear to be in the context of an intoxication or medical condition. Note that despite the
• 25 name "bi"polar, only a manic episode is required to make this diagnosis; and the patient's lack Send us your feedback at:
medFeedback@kaplan.com
• 26 of a history of depression does not change this diagnosis to a mood disorder NOS.
• 27 Please include OlD s2s135m

• 28
• 29
• 30 Exit


1
2
a Item: 20 of 44 <J C> il , . ~
Prev1ous Next Lab Values Notes Calculator
• 3
• 4
• 5 Explanation Rel<aps & Refs ~ Help 0 My Questions

• 6
The correct answer is A . This patient is most likely experiencing a manic episode that does 84% of students got this correct.
• 7
not appear to be in the context of an intoxication or medical condition. Note that despite the
• 8 Send us your feedback at:
name "bi"polar, only a manic episode is required to make this diagnosis; and the patient's lack
• 9 medFeedback@kaplan.com
of a history of depression does not change this diagnosis to a mood disorder NOS.
• 10 Please include QID s2s13Sm
• 11 Bipolar disorder, type II (choice B) , is characterized by hypomania and major depression.
• 12 This patient is grossly manic now, so BPD II is not a possibility.
• 13 Cyclothymic disorder (choice C) is characterized by hypomania and low-grade depression.
• 14 This patient meets criteria for a full-blown mania and would be diagnosed as bipolar disorder,
• 15 type I, rather than a cyclothymia.
• 16
Major depressive disorder with atypical features (choice D) is characterized by depressed
• 17
mood with "reverse neurovegetative symptoms" such as hypersomnia and increased appetite.
• 18
This patient is manic and is not depressed in presentation.
• 19
The patient is having a manic episode that is not occurring in the context of any apparent
• 21 intoxication or medical illness. Therefo re, she meets the criteria for bipolar disorder and does
• 22 not require a diagnosis of provisional mood disorder NOS (choice E) .
• 23 This patient's symptoms are consistent with a manic episode with possible psychotic features.
• 24 The patient, however, is not demonstrating the clinical history of a degenerative psychotic
• 25 illness occurring over months. Therefore, schizophrenia, chronic paranoid type (c hoice F) • is
• 26 incorrect.
• 27
• 28
• 29
• 30 Exit
• 1
• 2
• 3
A 23-year-old man is brought to the emergency department for evaluation of altered mental status and fevers. His friend is with him
• 4
and says that the man has been healthy recently but saw a psychiatrist several days ago for complaints of "hearing voices." He was
• 5
started on a new medication, but the friend does not know the name of it. He has a temperature of 40" C (104.1 • F). On examination
• 6 the patient is obtunded and not able to respond coherently to questioning. He has rigidity in all of his extremities. His creatinine
• 7 phosphokinase level is 8000 U/L. Which of the following is the most likely diagnosis?
• 8
• 9
• 10 0 A. Bacterial meningitis
• 11
0 B. Fungal meningitis
• 12
• 13 0 C. Metabolic encephalopathy
• 14 0 0 D. Neuroleptrc malignant syndrome
• 15
16 0 E. Status epilepticus

• 17
• 18
• 19 Explanation ReKaps & Refs @ Help 0 My Questions

• 20
• 21. The correct answer is D. This patient's cl inical presentation is suggestive of neuroleptic 97% of students got this correct.

• 22 malignant syndrome (NMS), which typically has an altered mental status, fever, dysautonomy,
Send us your feedback at:
and rigidity. There is also an elevated creatinine phosphokinase (CPK). and there can be a
• 23 medFeedback@kaplan.com
leukocytosis as well. The mechanism of NMS is unknown but because of the class of agents with
• 24 Please include QIO imq440
which NMS is associated, dopamine receptor blockade is central to most theories of its
• 25
pathogenesis. NMS is typically seen after administration of antipsychotic (neuroleptic)
• 26
medications. Treatment is cooling measures, hydration, dantrolene, bromocriptine, and
• 27 m"'iniAn"'n~A nf hAmnrlvn,.mi~ c:t,.hilitv
• 28
• 29
• 30 Exit


1
2
a Item: 21 of 44 <J C> il , . ~
Prev1ous Next Lab Values Notes Calculator
• 3
• 4
• 5 Explanation ReKaps & Refs f} Help 0 My Questions

• 6
• 7 The correct answer is D. This patient's clinical presentation is suggestive of neuroleptic 97% of students got this correct.

• 8 malignant syndrome (NMS), which typically has an aHered mental status, fever, dysautonomy,
Send us your feedback at:
and rigidity. There is also an elevated creatinine phosphokinase (CPK), and there can be a
• 9 medFeedback@kaplan.com
leukocytosis as well. The mechanism of NMS is unknown but because of the class of agents with
• 10 Please include QID imq440
which NMS is associated, dopamine receptor blockade is central to most theories of its
• 11
pathogenesis. NMS is typically seen after administration of antipsychotic (neuroleptic)
• 12
medications. Treatment is cooling measures, hydration, dantrolene, bromocriptine, and
• 13
maintenance of hemodynamic stability.
• 14
• 15 Bacterial meningitis (choice A) and fungal meningitis (choice B) can cause some of the
16 same symptoms but would not be precipitated by administration of a neuroleptic medication.

• 17 Meningitis does not elevate the CPK

• 18 A metabolic encephalopathy (choice C) would typically not be associated with fever or rigidity,
• 19 but could cause aHered mental status. A complete metabolic profile should be checked in
• 20 patients with altered mental status.
• 21.
Status epilepticus (choice E) , or prolonged seizure activity, can cause elevated CPKs; but
• 22
unless the underlying etiology is a CNS infection, status epilepticus is typically not associated
• 23 with fever.
• 24
• 25 Reviewed on 12/10112
• 26
• 27
• 28
• 29
• 30 Exit
• 1
• 2
• 3
A 28-year-old college student comes to the ER with the complaint of skin rashes that he developed 2 days ago. The patient was taken
• 4
to a psychiatrist 5 weeks ago because he was very loud and talkative. He stated that had not slept for the previous 4 days because he
• 5
had been making frequent trips to the library, but had been able to stay up all night to study without a problem. He knew he was going
• 6
to excel in his exams, because he recently had gained some power from God. The psychiatrist started him on an anticonvulsant that
• 7
the patient was told would help with his mood swings. His past history is significant for chickenpox at the age of 10. On examination
• 8 today, he is noted to have atypical targetoid lesions with focal confluence and detachment of the epithelium over about10o/o of his
• 9 body surface. Which of the following medications did this patient most likely take?
• 10
• 11
• 12 0 A. Lithium
• 13
0 B. Clonazepam
• 14
• 15 0 C. Gabapentin
• 16 (I 0 D. Lamotngme
• 17
• 18
0 E. Valproate

• 19
• 20
• Explanation Rel<aps & Refs @ Help 0 My Questions

The correct answer is D. Lamotrigine is approved as an adjunct in the treatment of refractory 72% of students got this correct.
• 23
• 24 seizures. The most serious side effect of lamotrigine is rash. When the dose is titrated up too
Send us your feedback at :
• 25 quickly, the potentially fatal Stevens-Johnson syndrome can occur. Carbamazepine is also medFeedback@kaplan.com
approved in the treatment of bipolar disorder and is also associated with Steven-Johnson
• 26 Please include QID Q0465m
syndrome, especially in the first 8 weeks of therapy.
• 27
• 28
• 29
• 30 Exit


1
2
a Item: 22 of 44 <J C> il , . ~
Prev1ous Next Lab Values Notes Calculator
• 3
• 4
• 5 The correct answer is D. Lamotrigine is approved as an adjunct in the treatment of refractory 72"!. of students got this correct

• 6 seizures. The most serious side effect of lamotrigine is rash. When the dose is titrated up too
Send us your feedback at:
• 7 quickly, the potentially fatal Stevens-Johnson syndrome can occur. Carbamazepine is also medFeedback@kaplan.com
• 8 approved in the treatment of bipolar disorder and is also associated with Steven-Johnson
Please include QID Q046Sm
• 9 syndrome, especially in the first 8 weeks of therapy.

• 10 Lithium (choice A) has been reported to cause Stevens-Johnson syndrome in a few patients
• 11 and it is commonly prescribed for bipolar disorder, but it is not an anticonvulsant. Lithium has
• 12 other side effects such as acne, weight gain, hypothyroidism, nephrotoxicity, and teratogenicity .
• 13 Before lithium is prescribed, a thyroid function test, renal function test, and pregnancy test
• 14 should be performed.
• 15
Clonazepam (choice B ) is a high-potency benzodiazepine approved for the treatment of
• 16
seizures. It is used as an adjunct in the treatment of manic patients or those with anxiety
• 17 disorders. The most common side effects include sedation and ataxia, as well as the potential
• 18 fo r dependency.
• 19
Gabapentin (choice C) is approved as an adjunct in the treatment of partial and generalized
• 20
seizures. It has also been used to treat bipolar disorders, pain syndromes, and anxiety

disorders. The most common side effects are somnolence, dry mouth, dizziness, and ataxia.

• 23 Valproate (choice E ) is an anticonvulsant that is also effective in the treatment of acute mania,
• 24 mixed states, and rapid-cycling bipolar disorder. Its most serious side effects include
• 25 hepatotoxicity and thrombocytopenia. In pregnancy, it can cause neural tube defects. II can
• 26 also lead to polycystic ovarian disease.
• 27
• 28
• 29
• 30 Exit
• 1
• 2
• 3
4 A 64-year-old man is undergoing chemotherapy and has occasional nausea and vomiting, which are treated with IV prochlorperazine.

After several days of therapy, the patient complains that he feels very restless and agitated and he cannot stop moving his legs. He
• 5
has never experienced these symptoms before and is concerned that they were caused by his recent therapy. Which of the following
• 6
medications would best treat his symptoms?
• 7
• 8
• 9 0 A. Chlorpromazine
• 10
• 11 O B. Haloperidol
• 12
8 o c. Lorazepam
• 13
0 0. Paroxetine
• 14
• 15 0 E. Sertraline
• 16
• 17
• 18 Explanation ReKaps & Refs 6 Help 0 My Questions
• 19
• 20 The correct answer is C. This patient has evidence of akathisia, the feeling of restlessness 91 % of students got this correct.

• 21 that sometimes occurs as a reaction to use of neuroleptic medications such as prochlorperazine


Send us your feedback at:
• 22 and haloperidol. medFeedback@kaplan.com
• 23 Chlorpromazine (choice A) and haloperidol (choice B) are both neuroleptic medications that Please include QIO Q0168m
• 24 would exacerbate akathisia rather than prevent it.
• 25
26 Paroxetine (choice D) and sertraline (choice E) are both antidepressant medications that are

specific serotonergic reuptake inhibitors. These agents have no effect on the prevention of
• 27
• 28
• 29
• 30 Exit


1
2
a Item: 23 of 44 <J C> il , . ~
Prev1ous Next Lab Values Notes Calculator
• 3
• 4
• 5 0 A. Chlorpromazine
• 6 0 B. Haloperidol
• 7
• 8 (I 0 C. Lorazepam
• 9 0 D. Paroxetine
• 10
0 E. Sertraline
• 11
• 12
• 13
• 14 Explanation ReKaps & Refs @ Help 0 My Questions

• 15
The correct answer is C. This patient has evidence of akathisia, the feeling of restlessness 91% of students got this correct
• 16
that sometimes occurs as a reaction to use of neuroleptic medications such as prochlorperazine
• 17 Send us your feedback at:
and haloperidol. medFeedback@kaplan. com
• 18
• 19 Chlorpromazine (choice A) and haloperidol (choice B) are both neuroleptic medications that Please include QID Q0168m
• 20 would exacerbate akathisia rather than prevent it
• 21 Paroxetine (choice D) and sertraline (choice E) are both antidepressant medications that are
• 22 specific serotonergic reuptake inhibitors. These agents have no effect on the prevention of
• 23 neuroleptic-induced akathisia.
• 24
• 25 Reviewed on 12/10112
• 26
• 27
• 28
• 29
• 30 Exit
• 1
• 2
• 3
A 35-year-old man is brought to the emergency clinic by his mother because of an episode of slurred speech associated with the
• 4 uncomfortable sensation that his tongue is thick and curling up. The episode started suddenly 30 minutes ago. The patient is noted to
• 5 be holding onto his tongue with his thumb and forefinger. His mother reports that the patient has had schizophrenia for 10 years and
• 6 consistently takes medications prescribed by his psychiatrist. Several days ago he ran out of one of his medications, but has
• 7 continued to take the others. Administration of which of the following should be included in the initial step in the management of this
• 8 patient?
• 9
• 10
• 11 0 A. Alprazolam
• 12 fl 0 B. Benztropme
• 13
• 14 0 C. Haloperidol
• 15 0 D. lorazepam
• 16
0 E. Olanzapine
• 17
• 18
• 19
• 20
Explanation Rel<aps & Refs 0 Help Q My Questions

• 21
The correct answer is B. This patient is experienc ing an acute dystonic reaction , which is a 65% of students got th is correct.
• 22
form of extrapyramidal side effect (EPS) associated with antipsychotic medications. These side
• 23 Send us your feedback a t:
effects are related to antagonism of dopamine receptors in the nigrostriatal pathway. The patient
medFeedback@kaplan.com
is likely taking a conventional antipsychotic agent plus a prophylactic anticholinergic agent
• 25 Please include QID Q0370m
(such as benztropine, diphenhydramine, or trihexyphenidyl). When he stopped taking the
• 26
anticholinergic , the dystonic reaction was more likely to occur. The appropriate initial
• 27 manaoement of th is oatient would include immediate IM administration of an anticholineraic
• 28
• 29
• 30 Exit
• 1
• 2
• 3
• 4 Explanation Rel<aps & Refs 6 Help Q My Questions

• 5
• 6 The correct answer is B. This patient is experiencing an acute dystonic reaction , which is a 65% of students got th is correct.

• 7 form of extrapyramidal side effect (EPS) associated with antipsychotic medications. These side
Send us your feedback at:
• 8 effects are related to antagonism of dopamine receptors in the nigrostriatal pathway. The patient
medFeedback@kaplan.com
is likely taking a conventional antipsychotic agent plus a prophylactic anticholinergic agent
• 9 Please include QID Q0370m
(such as benztropine, diphenhydramine, or trihexyphenidyt). When he stopped taking the
• 10
anticholinergic, the dystonic reaction was more likely to occur. The appropriate initial
• 11
management of this patient would include immediate IM administration of an anticholinergic
• 12
agent such as 2 mg of benztropine or 50 mg of diphenhydramine.
• 13
• 14 Alprazolam (choice A) is a short-acting, high-potency benzodiazepine commonly used for the
• 15 short-term management of anxiety. It would not be the appropriate treatment for an acute
• 16 dystonic reaction .
• 17 Haloperidol (choice C) is a conventional high-potency antipsychotic agent that would be likely
• 18 to cause an EPS such as dystonia. It would make the dystonic reaction worse .
• 19
Lorazepam (choice D) is, like alprazolam, a benzodiazepine It is commonly used in the acute
• 20
management of severe anxiety and agitation It would not be appropriate in the management of
• 21
acute dystonia.
• 22
• 23 Olanzapine (choice E) is an atypical antipsychotic agent Like the other atypical agents
(clozapine, risperidone, and quetiapine), it is associated with a lower incidence of EPS while
• 25 being more efficacious for the negative symptoms of schizophrenia. It would not be a treatment

26 for EPS.

• 27
• 28
• 29
• 30 Exit
• 1
• 2
• 3
4 A 42-year-old recently divorced woman started weekly cogn itive-behavioral therapy for symptoms of depression, including a 4-month

history of anhedonia, poor appetite, difficulty sleeping, low self-esteem, and poor concentration. After 8 weeks of cognitive-behavioral
• 5
therapy, her symptoms have not improved. She has lost weight, feels tired every day, and is feeling hopeless about her future. She is
• 6
referred for psychiatric consultation. Which of the following is the most appropriate next step in the management of this condition?
• 7
• 8
• 9 O A.ECT
• 10
• 11 0 B. Hospitalization

• 12 0 C. Increase cognitive behavioral therapy to twice per week


• 13
0 D. Reassurance and continue once-weekly cognitive behavioral therapy
• 14
• 15 0 0 E. Start a selective serotonin reuptake inhibitor
• 16
• 17
• 18 Explanation ReKaps & Refs 6 Help 0 My Questions
• 19
• 20 The correct answer is E. This woman is suffe ring from a major depressive disorder. She 88% of students got this correct.

• 21 started treatment with weekly cognitive-behavioral therapy, an appropriate treatment. After 8


Send us your feedback at:
• 22 weeks of psychotherapy, her depressive symptoms have worsened, but she is not acutely medFeedback@kaplan.com
• 23 suicidal. The most appropriate next step in treatment is to add medication, in this case, a
Please include QIO m001052
• 24 selective serotonin reuptake inhibitor.
• 25 Electroconvulsive therapy (ECT) (choice A) is a useful treatment for severe major depressive
• 26 disorder, but it is usually done if the depressive symptoms do not respond to multiple medication
• 27 trials .
• 28
• 29
• 30 Exit
• 1
• 2
• 3
0 E. Start a selective serotonin reuptake inhibitor
• 4
• 5
• 6
• 7 Explanation ReKaps & Refs t} Help Q My Questions
'

• 8
The correct answer is E. This woman is suffering from a major depressive disorder. She 88% of students got this correct
• 9
started treatment with weekly cognitive-behavioral therapy, an appropriate treatment. After 8
• 10 Send us your feedback at:
weeks of psychotherapy, her depressive symptoms have worsened, but she is not acutely medFeedback@kaplan.com
• 11
suicidal. The most appropriate next step in treatment is to add medication, in this case, a
• 12 Please include QID m001052
selective serotonin reuptake inhibitor.
• 13
• 14 Electroconvulsive therapy (ECD (choice A) is a useful treatment for severe major depressive
• 15 disorder, but it is usually done if the depressive symptoms do not respond to multiple medication
• 16 trials.
• 17 Hospitalization (choice B) is sometimes necessary in the treatment of major depressive
• 18 disorder, but because this woman is not acutely suicidal, it is appropriate to try further
• 19 outpatient treatment at this time .
• 20
Increasing the cognitive-behavioral therapy (choice C) to twice per week would not be
• 21
sufficient as a stand-alone treatment given that the patient's symptoms of depression are
• 22
worsening.
• 23
• 24 Reassurance and continuing the weekly cognitive-behavioral therapy (choice D) is not an

• 25 appropriate level of care, given this patient's symptoms of depression.

• 26
• 27
• 28
• 29
• 30 Exit
• 1
• 2
• 3
A 29-year-old, previously successful woman was climbing stairs in her new home about a month ago, when the house collapsed. She
• 4
was taken to the hospital with a fractured left femur. The psychiatry team was consulted because the patient complained of nightmares
• 5
and flashbacks and was afraid to go to sleep as a result. During the interview, she is tearful, and afraid that her fear of falling is
• 6
preventing her from fully participating in her rehabilitation , for wh ich she fears the team will discharge her from the hospital. Which of
• 7
the following is the most appropriate treatment?
• 8
• 9
• 10 0 A. Insight-oriented psychotherapy
• 11
12
0 B. No therapy because the patient needs to take responsibility for her treatment

• 13 0 C. Put a sitter to stay in the patienrs room 24 hours a day to calm her anx.iety
• 14
(j 0 0 . Start an antidepressant
• 15
• 16 0 E. Start benzodiazepines
• 17
• 18
• 19 Explanation Rel<aps & Refs 6 Help Q My Questions
• 20
• 21 The correct answer is D. The patient is having symptoms of posttraumatic stress disorder 54% of students got this correct.

• 22 (PTSD), as well as depressive symptoms. Sertraline is an antidepressant approved for treatment


Send us your feedback at:
• 23 of this disorder. Other antidepressants and anticonvulsants have also been shown to be effective medFeedback@kaplan.com
• 24 in the treatment of PTSD.
Please include QIO Q0463
• 25 Insight-oriented psychotherapy (choice A) is focused on getting insight into the underlying
unconscious conflicts on the basis of ex.ploration of transference feelings evoked during the
• 27 process. It is not suitable for the treatment of acute PTSD .
• 28
• 29
• 30 Exit


1
2
a Item: 26 of 44 <J C> il , . ~
Prev1ous Next Lab Values Notes Calculator
• 3
• 4
• 5
• 6 Explanation ReKaps & Refs ti Help 0 My Questions
• 7
• 8 The co rrect answer is D. The patient is having symptoms of posttraumatic stress disorder 54% of students got this correct
• 9 (PTSD), as well as depressive symptoms. Sertraline is an antidepressant approved for treatment
Send us your feedback at:
• 10 of this disorder. Other antidepressants and anticonvulsants have also been shown to be effective medFeedback@kaplan.com
• 11 in the treatment of PTSD.
Please include QID Q0463
• 12
Insight-oriented psychotherapy (choice A) is focused on getting insight into the underlying
• 13
unconscious conflicts on the basis of exploration of transference feelings evoked during the
• 14 process. It is not suitable for the treatment of acute PTSD.
• 15
16 No therapy (choice B) in a patient with obvious symptoms that are interfering with his or her

17 treatment and social functioning would be unacceptable and considered neglect.

• 18 A 24-hour watch by a sitter (choice C) would be indicated if the patient is actively suicidal and
• 19 has poor impulse control. Having a family member for support is encouraged, but enforcing a
• 20 regressed and dependent position by a 24-hour watch would not be appropriate.
• 21
Benzodiazepines (choice E) can be used fo r symptom relief in cases of acute stress
• 22 reaction , but they are not indicated as long-term treatment of PTSD because of their addictive
• 23 potential.
• 24
• 25 Reviewed on 12/10112

• 27
• 28
• 29
• 30 Exit
• 1
• 2
• 3
A medical student is interviewing a 78-year-old man who was brought from the nursing home to the emergency department. The
• 4
student has been talking to the man for almost 30 minutes and has obtained very little information. Every time he asks a question, the
• 5
patient starts talking and goes into unnecessary details, eventually answering the question but only after he has told his entire story to
• 6
the medical student. The student is becoming frustrated with the patient because there are two other patients waiting to be interviewed.
• 7
This patient's speech is an example of which of the following?
• 8
• 9
• 10 (I 0 A. C1rcumstanllalrty
• 11
0 B. Flight of ideas
• 12
• 13 0 C. Loose associations
• 14
0 0 . Tangentiality
• 15
• 16 0 E. Word salad
• 17
• 18
• 19 Explanation Rel<aps & Refs ti Help Q My Questions

• 20
• 21 The correct answer is A. Circumstantiality refers to speech that is delayed from reaching the 31% of students got this correct.

• 22 point, characterized by over-inclusion of details. Eventually , it does get to the original goal.
Send us your feedback at:
• 23 Flight of ideas (c hoice B) is described as a rapid shift of ideas with some connection. It is medFeedback@kaplan.com
• 24 seen in manic patients (e.g., "I wore a green T-shirt today" ... "green is the color of grass" ... "it Please include QIO Q0083m
• 25 is the jersey of the Holland football team" ... "I went to Holland last year," etc.) .
• 26
Loose associations (ch oice C) consist of ideas that are disconnected and seem to jump from
• 71
• 28
• 29
• 30 Exit


1
2
a Item: 27 of 44 <J C> il , . ~
Prev1ous Next Lab Values Notes Calculator
• 3
0 D. Tangenllahty
• 4
• 5 0 E. Word salad
• 6
• 7
• 8 Explanation ReKaps & Refs 0 Help 0 My Questions
• 9
• 10 The c orrect answer is A. Circumstantiality refers to speech that is delayed from reaching the 31% of students got this correcL
• 11 point, characterized by over-inclusion of details. Eventually, it does get to the original goaL
Send us your feedback at:
• 12 medFeedback@kaplan.com
Flight of ideas (choice B) is described as a rapid shift of ideas with some connection. It is
• 13 seen in manic patients (e.g ., "I wore a green T-shirt today" ... "green is the color of grass" ... "it Please include QID Q0083m
• 14 is the jersey of the Holland football team" ... "I went to Holland last year," etc.).
• 15
• 16 loose associations (choice C) consist of ideas that are disconnected and seem to jump from
one topic to an unconnected topic. loose associations are one of the characteristic signs of a
• 17
primary thought disorder, such as schizophrenia (e g , "I wore a green T-shirt today" ... "my
• 18
grandma made me fruit salad today" ... "I spoke to my best friend yesterday" ... "I like your tie,"
• 19
etc.).
• 20
• 21 Tangentiality (choice D) is the inability to provide a goal-directed association to the given
• 22 thought. The patient never gets to the desired goal from the starting point of the discussion.
• 23 Word salad (choice E) is a total mixture of words that make no sense at all.
• 24
• 25 Reviewed on 12/10112
• 26
• 71
• 28
• 29
• 30 Exit


1
2
a Item: 28 of 44 <J C> il , . ~
Prev1ous Next Lab Values Notes Calculator
• 3
• 4 A 20-year-old woman has a history of repeated admissions for sudden abdominal pain. No known etiology has been found for this
• 5 pain despite intensive metabolic, infectious, and endocrine evaluations. The patient has also had repeated imaging studies of her
• 6 abdomen and one exploratory laparotomy. She comes to the emergency department complaining of similar pain and is admitted to the
• 7 hospital for management. A social-work consult is called and reveals that the patient is a single mother with two small children. Now on

• 8 the inpatient noor. the patient is relaxed. Extensive imaging and laboratory studies are all normal. Which of the following is the most

9 likely diagnosis?

• 10
• 11
0 A. Body dysmorphic disorder
• 12
• 13 0 B. Conversion disorder
• 14 0 0 C. Factrtrous drsorder
• 15
16 0 0 . Hypochondriasis

• 17 0 E. Malingering
• 18
• 19
• 20 Explanation Rel<aps & Refs 8 Help Q My Questions
• 21
• 22 The correct answer is C . Factitious disorder is a somatoform disorder in which symptoms are 61% of students got th is correct.
• 23 produced by patients in order for them to assume the "sick" role .
Send us your feedback at :
• 24 medFeedback@kaplan .com
Body dysmorphic disorder (choice A) is the belief that a body part is abnormal, misshapen, or
• 25
defective. This does not pertain to this patient's symptoms. Please include QIO s2s140m
• 26
• 27 Conversion disorder (choice B) is the experience of one or more neurologic symptoms in

• 29
• 30 Exit


1
2
a Item: 28 of 44 <J C> il , . ~
Prev1ous Next Lab Values Notes Calculator
• 3
• 4 0 D. Hypochondriasis
• 5
0 E. Malingering
• 6
• 7
• 8
• 9
Explanation ReKaps & Refs 0 Help 0 My Questions

• 10
The c orrect answer is C . Factitious disorder is a somatoform disorder in which symptoms are 61% of students got this correcL
• 11
produced by patients in order for them to assume the "sick" role.
• 12 Send us your feedback at:
• 13 Body dysmorphic disorder (choice A) is the belief that a body part is abnormal, misshapen, or medFeedback@kaplan.com

• 14 defective. This does not pertain to this patienrs symptoms. Please include QID s2s 140m
• 15 Conversion disorder (choice B) is the experience of one or more neurologic symptoms in
• 16 response to a psychologic stressor. This patienrs symptoms do not pertain to the nervous
• 17 system. The patient is not conscious about the symptoms she produces, nor does she have a
• 18 motivation or intention for it.
• 19
Hypochondriasis (choice D) is the belief that one has a specific disease. This patient does not
• 20
demonstrate this belief insofar as offe ring an etiology fo r her presenting symptoms.
• 21
• 22 Factitious disorder differs from malingering (choice E) in that it is exclusively the production
• 23 of the "sick" role that motivates patients with factitious disorder, and not some other secondary
• 24 gain. A patient who is malingering produces the symptom intentionally and has a motive (e.g.,
• 25 money, litigation, escape from responsibilities).

• 26
• 27

• 29
• 30 Exit
• 1
• 2
• 3 xu-vF•ar--mr woman presents to the outpatient clinic with her daughter. She reports that she has had d ng
• 4 few months, including taking 45 to 60 minutes to fall asleep, frequent midnight awakenings to use the bathroom and stretch her legs,
• 5 and sleeping on average 5 to 6 hours per night. She finds that she needs to take a short nap during the day. She reports that her
• 6 husband of 55 years died 1 year ago, and she misses him very much. She reports that she lives alone and has some mild memory
• 7 loss. She denies change in her appetite or weight. She reports an active social life with friends from her retirement community and her
• 8 family. She denies a history of snoring at night. Her physical examination is unremarkable. What is the most likely diagnosis?
• 9
• 10
• 11 (I 0 A. Age-related sleep changes
• 12 0 B. Bereavement
• 13
0 C. Early Alzheimer disease
• 14
• 15 0 0 . Major depressive disorder
• 16
0 E. Restless legs syndrome
• 17
• 18
• 19
20
Explanation ReKaps & Refs @ Help 0 My Questions

• 21
The correct answer is A. Older individuals require less nighttime sleep, take longer to fall 90% of students got this correct.
• 22
asleep, often nap during the day, and have more middle of the night awakenings than they did
• 23 when they were younger. Sleep changes can be attributed to changes such as decrease in
Send us your feedback at:
medFeedback@kaplan.com
• 24 percentage of total REM and stage 3 and 4 NREM sleep thus making it difficult for them to have
• 25 a deep sleep resulting to a less relaxing sleep. These age-related changes are normal and do Please include QIO mOOlOSO

• 26 not indicate a sleep disorder.


• 27 n ... - .............................. ,_..__: __ " ' - - - ....................... :•&-. ......................................... c : ...................... ; .............. - - - - ................. ~- .... ...... ...
• 28
• 29
• 30 Exit
• 1
• 2
• 3
• 4
• 5
Explanation Rel<aps & Refs @ Help 0 My Questions

• 6
The correct answer is A. Older individuals require less nighttime sleep, take longer to fall 90% of students got this correct.
• 7
asleep, often nap during the day, and have more middle of the night awakenings than they did
• 8 Send us your feedback at:
when they were younger. Sleep changes can be attributed to changes such as decrease in medFeedback@kaplan.com
• 9 percentage of total REM and stage 3 and 4 NREM sleep thus making it difficun for them to have
• 10 a deep sleep resuning to a less relaxing sleep. These age-related changes are normal and do
Please include QID mOOlOSO

• 11 not indicate a sleep disorder.


• 12
Bereavement (choice B) can present with symptoms of insomnia and poor sleep. The sleep
• 13
changes associated with normal bereavement do not usually last longer than 2 months after the
• 14
death of a loved one.
• 15
• 16 Ear1y Alzheimer disease (choice C) can present with symptoms of anered sleep, but this
• 17 woman does not present with other symptoms (except for mild memory loss) to qualify for this
• 18 diagnosis.
• 19 Major depressive disorder (choice D) can be associated with sleep changes, such as
• 20 difficulty falling and staying asleep. This woman does not meet the criteria for a major
• 21 depressive disorder.
• 22
Restless legs syndrome (choice E) is a disorder in which one has the urge to kick or move
• 23
one's legs that worsens in the evening. It can be very disruptive of sleep and most often occurs
• 24
in individuals older than age 40 years. Aside from needing to stretch her legs, this woman does
• 25
not meet the criteria for the disorder.
• 26
• 27
• 28
• 29
• 30 Exit
• 1
• 2
• 3
A 72-year-old woman is brought to the emergency department by her daughter, who is concerned her mother had a "stroke."
• 4
According to the daughter, the woman woke up that morning not knowing where she was. She was confused and did not remember
• 5
what had happened the day before. Her history is significant for hypertension, diabetes mellitus type 2, angina, and anxiety disorder;
• 6 and she takes atenolol, hydrochlorothiazide, metformin, and alprazolam. On physical examination, she is found to have a BP of 128/76
• 7 mm Hg, PRof 86/min, RR of 14/min, and temperature of 98.8 •F (37.1 •c). Neurologic examination is unremarkable. EKG and ABG
• 8 are found to be normal. Which of the following should be the next step?
• 9
• 10
• 11 0 A. Carotid duplex scanning
• 12
0 B. Discontinue atenolol
• 13
• 14 0 C. Echocardiogram
• 15 0 D. Order troponins
• 16
• 17 f) 0 E. Stop alprazolam
• 18
• 19
• 20 Explanation Rel<aps & Refs @ Help Q My Questions

• 21
• 22 The correct answer is E. Alprazolam belongs to the group of short-acting benzodiazepines. 68% of students got this correct.

• 23 Even though it has a short half-life, it can produce confusion, disinhibition, and amnestic
Send us your feedback at:
problems such as blackouts in the elderly population. Thus, benzodiazapines are not
• 24 medFeedback@kaplan .com
recommended in the elderly population, and if needed, should be given in lower doses than what
• 25 Please include QID Q0076m
is prescribed to the younger population.
• 26
• 27 Currently, the patient has stable vital signs and there is no abnormality detected on neurologic
• 28

Exit


1
2
a Item: 30 of 44 <J C> il , . ~
Prev1ous Next Lab Values Notes Calculator
• 3
• 4
0 D. Order troponins
• 5
• 6 f) 0 E. Stop alprazolam
• 7
• 8
• 9 Explanation ReKaps & Refs 6 Help 0 My Questions
• 10
• 11 The correct answer is E. Alprazolam belongs to the group of short-acting benzodiazepines. 68% of students got this correct.
• 12 Even though it has a short half-life, it can produce confusion, disinhibition, and amnestic
Send us your feedback at:
• 13 problems such as blackouts in the elderly population. Thus, benzodiazapines are not medFeedback@kaplan.com
• 14 recommended in the elderly population, and if needed, should be given in lower doses than what
Please include QID Q0076m
• 15 is prescribed to the younger population.
• 16 Currently, the patient has stable vital signs and there is no abnormality detected on neurologic
• 17 examination, so previous acute confusion is most likely due a side effect of aprazolam. Neither
• 18 carotid duplex scanning (choice A) nor an echocardiogram (choice C) would be the
• 19 appropriate next step in the management because this patient is having these symptoms due to
• 20 the benzodiazepines, which should be stopped first
• 21
Atenolol (choice B) has no relation with the symptoms described, so it need not be
• 22
discontinued.
• 23
• 24 Ordering troponins (choice D) should not be the next step because there is nothing in the stem
• 25 to lead to a diagnosis of Coronary Artery Disease (CAD). Even the EKG is normal.

• 26
• 27
• 28

Exit
• 2
• 3
• 4 A 31 -year-old man with schizoaffective disorder comes with his fam ily to the psychiatrist for follow-up. He has been stable and
• 5 compliant with his medication. He is very concerned, however, about the 15 lb he has gained in the past 3 months, despite trying to
• 6 watch his diet and increase his exercise. His wife tells the physician that she heard about a new medication that is available for the
• 7 treatment of schizophrenia and is seemingly weight-neutral. She requests that her husband be switched to this new drug. To wh ich of
• 8 the following antipsychotic medications is she most likely referring?
• 9
• 10
11 0 A. Clozapine

• 12 0 B. Olanzapine
• 13
0 C. Quetiapine
• 14
• 15 0 0 . Risperidone
• 16 (I 0 E. Zlprastdone
• 17
• 18
• 19
20
Explanation Rel<aps & Refs ti Help Q My Questions

• 21
The correct answer is E . Of the antipsychotics listed, ziprasidone is the only one that is 42"/o of students got this correct.
• 22
weight-neutral. It can initially produce gastrointestinal symptoms and needs to be taken with
• 23 meals. Arrhythm ia (QT prolongation) is a unique side effect of this particular antipsychotic drug.
Send us your feedback at:
medFeedback@kaplan.com
• 24 Minor QT prolongation may occur more frequently; clinically relevant prolongation (>500 msec)
• 25 Please include QIO s2s 154m
is rare .
• 26
Clozapine (c hoice A) is known to cause significant weight gain. Other serious side effects
• 27
include seizures and agranulocytosis.
• 28
• 29
• 30
Exit
• 31.
• 2 a Item: 31 of 44 <J C> il , . ~
• 3 Prev1ous Next Lab Values Notes Calculator

• 4
• 5 0 C. Quetiapine
• 6
0 D. Risperidone
• 7
• 8 f) 0 E. Z1pras1done
• 9
• 10
• 11 Explanation ReKaps & Refs 0 Help 0 My Questions
• 12
• 13 The correct answer is E . Of the antipsychotics listed, ziprasidone is the only one that is 42% of students got tflis correct.

• 14 weight-neutral. It can initiaOy produce gastrointestinal symptoms and needs to be taken with
Send us y our feedback at:
• 15 meals. Arrhythmia (QT prolongation) is a unique side effect of this particular antipsychotic drug.
medFeedback@kaplan.com
• 16 Minor QT prolongation may occur more frequently; clinically relevant prolongation (>500 msec)
Please include QID s2s 154m
• 17 is rare .
• 18 Clozapine (choice A) is known to cause significant weight gain. Other serious side effects
• 19 include seizures and agranulocytosis
• 20
Olanzapine (choice B) is associated with significant increase in weight and diabetes.
• 21
• 22 Quetiapine (choice C) is also associated with an inc rease in weight that is somewhat less than
• 23 that of clozapine or olanzapine. Quetiapine has the lowest extrapyramidal side effects.
• 24
Risperidone (choice D) also causes weight gain, though not as much as clozapine or
• 25 olanzapine.
• 26
• 27
• 28
• 29
• 30
Exit
• 31.
• 3
• 4
• 5 A 10-year-old boy comes to the physician with his parents at the request of school authorities. In the past couple of months the
• 6 teachers have noticed that he has become more fidgety in class and at times coughs. He also makes sudden arm movements to touch
• 7 his peers in the middle of conversation. He also blinks often, and this has recently worsened. He says he has a feeling of tightness
• 8 that is relieved by arm extension, and a burning feeling in his eye before an eye blink. The main reason for the school's calling the
• 9 parents and insisting on a medical evaluation is the boy's sudden screams that distract and scare the other students in class. He

10 seems very upset when it occurs and admits that he is aware that other children make fun of him and avoid him because of it but he

cannot control it. He is successful academically and is very neat, and there are no other behavioral or emotional problems. What other
• 11
psychiatric disorder can be associated with this patienfs condition?
• 12
• 13
• 14
(I O A. ADHD
• 15
• 16 0 B. Autistic disorder
• 17 0 C. Absence seizure
• 18
0 D. Huntington disease
• 19
• 20 0 E. Oppositional defiant disorder
• 21
• 22
• 23 Explanation Rel<aps & Refs @, Help 0 My Questions
• 24
• 25 The correct answer is A . This boy most likely has Tourelle disorder. II is most common in 92% of students got this correct.
• 26 boys. Multiple motor tics (sudden arm movements, blinking) and vocal tics (coughing,
Send us your feedback a t:
• 27 screaming) are present during the illness many times a day, every day, for more than a year.
medFeedback@kaplan.com
• 28 Premonitory feelings or sensations (a feeling of tightness that is relieved by arm extension, or a
Please includP rnn c'?c 1 ,;7rn
• 29 hum inn SP.nsAiinn in lhP. P.VP. hP.fnrP. An P.VP. hlink) mAv nrP.c:P.ClP. mnlnr ::mel vnc:AIIic:s in mAnv

Exit
• 3 a Item: 32 of 44 <J C> il , . ~
• 4 Prev1ous Next Lab Values Notes Calculator

• 5
• 6
• 7 Explanation ReKaps & Refs @ Help 0 My Questions
• 8
• 9 The correct answer i s A . This boy most likely has Tourette disorder. It is most common in 92% of students got this correct.

• 10 boys. Multiple motor tics (sudden arm movements, blinking) and vocal tics (coughing,
Send us your feedback at:
• 11 screaming) are present during the illness many times a day, every day, for more than a year. medFeedback@kaplan.com
• 12 Premonitory feelings or sensations (a feeling of tightness that is relieved by arm extension, or a
Please include QID s2s167m
• 13 burning sensation in the eye before an eye blink) may precede motor and vocal tics in many
• 14 patients. The disturbance is not caused by a general medical condition or effects of substance.

15 Coprolalia and echolalia typically occur later. Tourette syndrome is associated with ADHD and

OCD in one-third of these patients.
• 16
• 17 Autistic disorder (choice B) is a pervasive developmental disorder starting in ear1y childhood .
• 18 It is manifested by qualitative impairment in social interaction, communication, and restricted
• 19 repetitive and stereotyped patterns of behavior or interests. Delay or abnormal functioning with
• 20 onset before 3 years of age is seen in the areas of social interaction, language, and
• 21 communication or symbolic play.
• 22 Absence seizure (choice C) is a kind of generalized seizure in which the patient may present
• 23 with a history of staring spells, not blinking Moreover, absence seizure is associated with a
• 24 decline in school performance. This patient, however, is academically successful.
• 25
Huntington disease (choice D ) is inherited in an autosomal-dominant way. It begins in
• 26
mid-adult life and is characterized by movement disorder and intellectual decline. Involuntary
• 27
choreatic movements affect limbs or trunk and take the form of rapid , jerky movements.
• 28
Intellectual decline usually progresses into dementia of the subcortical type. This patient is
• 29

Exit
• 3 a Item: 32 of 44 <J C> il , . ~
• 4 Prev1ous Next Lab Values Notes Calculator

• 5
patients. The disturbance is not caused by a general medical condition or effects of substance.
• 6
Coprolalia and echolalia typically occur later. Tourette syndrome is associated with ADHO and
• 7
OCO in one-third of these patients.
• 8
• 9 Autistic disorder (choice B) is a pervasive developmental disorder starting in early childhood.
• 10 II is manifested by qualitative impairment in social interaction, communication, and restricted
• 11 repetitive and stereotyped patterns of behavior or interests. Delay or abnormal functioning with

• 12 onset before 3 years of age is seen in the areas of social interaction, language, and
communication or symbolic play.
• 13
• 14 Absence seizure (choice C) is a kind of generalized seizure in which the patient may present
• 15 with a history of staring spells, not blinking. Moreover, absence seizure is associated with a
• 16 decline in school performance. This patient, however, is academically successful.
• 17
Huntington disease (choice D) is inherited in an autosomal-dominant way. II begins in
• 18 mid-adu~ life and is characterized by movement disorder and intellectual decline. Involuntary
• 19 choreatic movements affect limbs or trunk and take the form of rapid, jerky movements.
• 20 Intellectual decline usually progresses into dementia of the subcortical type. This patient is
• 21 doing well academically, and the sudden movement he has is due to a motor tic, not a chorea .
• 22
Oppositional defiant disorder (choice E) is a disorder of childhood consisting of a pattern of
• 23
negativistic, hostile, and defiant behavior lasting at least 6 months, during which at least four of
• 24
the following symptoms must be present: loss of temper, arguments with adu~s. blaming others
• 25
for own mistakes, deliberately annoying others, and refusing to comply with rules. The present
• 26
patient's vocal and motor tics are not deliberate.
• 27
• 28
• 29

Exit
• 4
• 5
• 6 A 34-year-old woman in her first trimester of pregnancy is evaluated by a psychiatrist. She recently started saying that she hears
• 7 voices telling her to kill herself because she does not deserve to live. When questioned, she reports that she feels extremely
• 8 depressed, guilty, and worthless. Her husband says that before this, she was a stable, content woman who was very happy about
• 9 having this ch ild. Which of the following is the most appropriate treatment at this time?
• 10
• 11
• 12
0 A. Clonazepam

• 13 (I 0 B. Electroconvulsrve therapy (ECT)


• 14
0 C. Fluoxetine
• 15
• 16 0 0 . Lithium
• 17 0 E. Psychotherapy
• 18
• 19
• 20
Explanation ReKaps & Refs G Help Q My Questions
• 21
• 22
The correct answer is B . Electroconvulsive therapy (ECT) has been safely used in pregnant 63% of students got this correct.
• 23 women. In case of psychotic depression with increased risk fo r suicide, the situation requires
• 24 expeditious treatment to protect the mother and fetus, and ECT is the treatment of choice.
Send us your feedback at:
medFeedback@kaplan.com
• 25
Clonazepam (choice A) is not indicated as a primary treatment of depression with psychotic Please include QJO Q0069m
• 26
features. During the first trimester of pregnancy, the use of benzodiazepines increases the risk
• 27
of cleft palate 10-fold .
• 28
• 29 Fluoxetine (choice C) , along with tricyclic antidepressants, is currently considered the safest
~"in 11"1 tn 11<:0 r'111ri nn nrOI"'n!:lnl"\1 In tho ,....~co nf n c \tt"hntil" r4onroccinn
• 30 hn\vouor rnnnnthoron\1 ,..,jth

• 31
• 32
Exit
• 33
• 4 a Item: 33 of 44 <J C> il , . ~
• 5 Prev1ous Next Lab Values Notes Calculator

• 6
• 7 0 E. Psychotherapy
• 8
• 9
• 10 Explanati on ReKaps & Refs G Help Q My Questions
• 11
• 12 The correct answer is B. Electroconvulsive therapy (ECT) has been safely used in pregnant 63% of students got this correcL
• 13 women. In case of psychotic depression with increased risk for suicide, the situation requires
Send us your feedback at:
• 14 expeditious treatment to protect the mother and fetus, and ECT is the treatment of choice. medFeedback@kaplan.com
• 15
Clonazepam (choice A) is not indicated as a primary treatment of depression with psychotic Please include QID Q0069m
• 16
features. During the first trimester of pregnancy, the use of benzodiazepines increases the risk
• 17
of cleft palate 10-fold.
• 18
• 19 Fluoxetine (choice C) • along with tricyclic antidepressants, is currently considered the safest
• 20 drug to use during pregnancy. In the case of psychotic depression, however, monotherapy with
21 fluoxetine would not be sufficient and would require longer inpatient treatment.

• 22 Lithium use (choice D) during the first trimester of pregnancy increases the risk for Ebstein
• 23 anomaly 10- to 20-fold. In addition, lithium has not proven to be an efficient antidepressant in
• 24 the treatment of psychotic depression It is primarily used as a mood stabilizer.
• 25
Psychotherapy (choice E) is an important treatment option for helping women understand the
• 26 conflictual feelings that arise during pregnancy. However, given the psychotic symptoms and
• 27 severity of depression in this case, pharmacotherapy or ECT are the treatments of choice.
• 28
• 29
• 30
• 31
• 32
Exit
• 33
• 5
• 6
• 7
A 17-year-old adolescent is brought to the hospital by police after being slopped for driving too slowly. He denies any use of alcohol,
• 8 but seems slow. He is laughing inappropriately, and has a dry mouth. His gail is somewhat slow and his coordination is impaired. His
• 9 eyes are red , which he explains as being due to pollen allergy. Which of the following drugs will most likely appear on a urine drug
• 10 screen?
• 11
• 12
• 13 0 A. Amphetamines
• 14
0 B. Benzodiazepines
• 15
• 16 0 C. Cocaine
• 17 0 D. Opiates
• 18
• 19 (I 0 E. Tetrahydrocannabinol
• 20
• 21
• 22 Explanation ReKaps & Refs 6 Help 0 My Questions

• 23
24 The correct answer is E. Tetrahydrocannabinol (cannabis) intoxication is usually 51% of students got this correct.

25 characterized by:
• Send us your feedback at:
• 26 • Sensitivity to external stimuli medFeedback@kaplan.com

• 27 • Subjective slowing down Please include QID Q047lm


• 28 • Impairment of motor skills (including problems with operation of motor vehicles)
• 29 • Conjunctival injection
• 30 • Dry mouth
• 31 • Tachvcardia

Exit
• 5
• 6
• 7 ReKaps & Refs

• 8
• 9 The correct answer is E. Tetrahydrocannabinol (cannabis) intoxication is usually 51 % of students got this correct

• 10 characterized by:
Send us your feedback at:
• 11 • Sensitivity to external stimuli medFeedback@kaplan.com
• 12 • Subjective slowing down Please include QID Q047lm
• 13 • Impairment of motor skills (including problems with operation of motor vehicles)
• 14 • Conjunctival injection
• 15 • Dry mouth
• 16 • Tachycardia
• 17 • Increased appetite
• 18
Amphetamine intoxication (choice A) produces maladaptive behavior, as well as two or more of
• 19 the following : pupillary dilatation, weight loss, perspiration, changes in blood pressure,
• 20 psychomotor changes, changes in heart rate, muscular weakness, seizures, and coma.
• 21
Benzodiazepine intoxication (choice B) usually results in behavioral disinhibition, euphoria,
• 22
slurred speech, incoordination, nystagmus, memory impairment, and stupor or coma.
• 23
• 24 Cocaine intoxication (choice C) includes maladaptive behavior (euphoria, hypervigilance,
• 25 anxiety, or anger), tachycardia, elevated blood pressure, nausea, weight loss, pupillary
• 26 dilatation, psychomotor changes, seizure, confusion, muscular weakness, and coma.
• 27
Opiate intoxication (choice D) involves maladaptive behavior, pupillary constriction, slurred
• 28 speech, impaired attention and memory, and drowsiness or coma.
• 29
• 30
• 31

Exit
• 6
• 7
• 8 A 21-year-old man drops out of college because of poor performance. On questioning, he explains that he is afraid of having another
• 9 episode of sudden discomfort in which he develops a fear of losing control. He had four such attacks in the last month. He became so
• 10 afraid that he could not concentrate in class and is no longer willing to sit in class because he would not be able to leave. Which of the
• 11 following is the most likely diagnosis?
• 12
• 13
• 14 0 A.AOHD
• 15 (I 0 B. Agoraphobia
• 16
0 C. Mental retardation
• 17
• 18 0 D. Social phobia
• 19
0 E. Somatization disorder
• 20
• 21
• 22
• 23
Explanation ReKaps & Refs f) Help Q My Questions

• 24
The correct answer is B . This patient has agoraphobia Although this condition was originally 80% of students got this correct.
• 25
defined as the fear of open spaces or of the marketplace, a more functional, modern definition
• 26 is a fear of panic attacks (episodes of sudden discomfort in which patients develop a fear of
Send us your feedback at:
medFeedback@kaplan.com
• 27 losing control) in situations from which it would be difficult to gracefully remove oneself. Like the
• 28 man in the question stem, a patient's fear may involve multiple settings and may progress to the Please include QJO Q0643m

• 29 point of markedly hampering daily functioning. The panic attacks may or may not actually have
• 30 been experienced in the past in the particular settings that are of concern to the patient. Some
• 31 cases resolve spontaneously; others pursue a waxing and waning course. Behavioral therapy is
• 32
• 33
• 34
Exit
• 35
• 6 a Item: 35 of 44 <J C> il , . ~
• 7 Prev1ous Next Lab Values Notes Calculator

• 8
• 9 The correct answer is B. This patient has agoraphobia. Although this condition was originally 80% of students got this correct.

10 defined as the fear of open spaces or of the marketplace, a more functional, modern definition
• Send us your feedback a t:
is a fear of panic attacks (episodes of sudden discomfort in which patients develop a fear of
• 11 medFeedback@kaplan.com
losing control) in situations from which it would be difficult to gracefully remove oneself. Like the
• 12 Please include QID Q0643m
man in the question stem, a patienrs fear may involve multiple settings and may progress to the
• 13
point of markedly hampering daily functioning. The panic attacks may or may not actually have
• 14
been experienced in the past in the particular settings that are of concern to the patient. Some
• 15
cases resolve spontaneously; others pursue a waxing and waning course. Behavioral therapy is
• 16 used to encourage patients to modify their activities. Antidepressants are helpful in patients with
• 17 coexisting depression.
• 18
19 Patients with ADHD (choice A) have difficulty regulating their attention and inhibiting their

attention to nonrelevant stimuli. The words "could not concentrate in class" are used as a
• 20
distractor, because all the symptoms indicate toward agoraphobia.
• 21
• 22 Mental retardation (choice C) is not the diagnosis. The only clue that could mislead a student
• 23 into picking this option is the fact that the patient dropped out of college due to poor
• 24 performance . But on reading the stem further, it is clear that the dropping out and poor
• 25 performance are due to the agoraphobia, and not retardation .
• 26 Social phobia (choice D) specifically refers to cl inically significant anxiety induced by social
• 27 or performance situations. Although the overlap with agoraphobia should be obvious. a tip-off
• 28 that that the answer to a test question is agoraphobia rather than social phobia is the use of the
• 29 term "panic attacks" (real life is fuzzier) .
• 30
Somatization disorder (choice E) is a severe psychiatric disorder in which a patient has had
• 31
many physical complaints over a period of years that do not have adequate anatomic pathologic
• 32
• 33
• 34
Exit
• 35
• 6 a Item: 35 of 44 <J C> il , . ~
• 7 Prev1ous Next Lab Values Notes Calculator

• 8 medFeedback@kaplan .com
losing control) in situations from which it would be difficult to gracefully remove oneself. Like the
• 9 Please include QID Q0643m
man in the question stem, a patient's fear may involve multiple settings and may progress to the
• 10
point of markedly hampering daily functioning. The panic attacks may or may not actually have
• 11
been experienced in the past in the particular settings that are of concern to the patient. Some
• 12 cases resolve spontaneously; others pursue a waxing and waning course. Behavioral therapy is
• 13 used to encourage patients to modify their activities. Antidepressants are helpful in patients with
• 14 coexisting depression.
• 15
Patients with ADHD (choice A) have difficulty regulating their attention and inhibiting their
• 16
attention to nonrelevant stimuli. The words "could not concentrate in class" are used as a
• 17
distractor, because all the symptoms indicate toward agoraphobia.
• 18
• 19 Mental retardation (choice C) is not the diagnosis. The only clue that could mislead a student
• 20 into picking this option is the fact that the patient dropped out of college due to poor
• 21 performance. But on reading the stem further, it is clear that the dropping out and poor
• 22 performance are due to the agoraphobia, and not retardation.
• 23 Social phobia (choice D) specifically refers to clinically significant anxiety induced by social
• 24 or performance situations. Although the overlap with agoraphobia should be obvious, a tip-off
• 25 that that the answer to a test question is agoraphobia rather than social phobia is the use of the
• 26 term "panic attacks" (real life is fuzzier)
• 27
Somatization disorder (choice E) is a severe psychiatric disorder in which a patient has had
• 28
many physical complaints over a period of years that do not have adequate anatomic pathologic
• 29
explanations.
• 30
• 31
• 32
• 33
• 34
Exit
• 35
• 7 a Item: 36 of 44 <J C> il , . ~
• 8 Prev1ous Next Lab Values Notes Calculator

• 9
A 65-year-old woman comes to a primary care physician with a complaint of poor memory. Her husband died 6 months ago and she
• 1.0
has recently decreased her church involvement and had less interest in her hobbies. She compla ins of feeling tired and worried , and
• 1.1.
has difficulty with attention and memory. She has a history of a major depressive episode approximately 10 years ago, which required
• :12
hospitalization. She was treated with an antidepressant for 2 years until it was tapered and discontinued. She currently denies
• 1.3 depressed mood. She states that her mother died of Alzheimer disease, and that she fi nds her memory loss particularly worrisome in
• 1.4 light of this family history. Which of the following factors would be more likely if this patient has a pseudodementia of depression
• 1.5 rather than a true dementia?
• 1.6
• 1.7
• 1.8 0 A. The patient appears unconcerned
• 1.9
(I 0 B. The patient seems concerned due to memory loss
• 20
• 21. 0 C. The patient has more difficulty with recent memory than remote memory
• 22 0 D. The patient tends to conceal the difficulty
• 23
24 0 E. The patient tries hard to answer questions about orientation and registration

• 25
• 26
• 27 Explanation Rel<aps & Refs @ Help 0 My Questions

• 28
The correct answer is B. Distinguishing a pseudodementia of depression from a true 69% of students got this correct.
• 29
dementia is an important task requiring examination of subtle aspects of the mental status
• 30 Send us your feedback at:
examination because treatment for the two conditions would be very different, and memory loss
• 31. medFeedback@kaplan.com
in th is woman should not be automatically presumed to be related to a primary dementia. In
• 32 Please include QIO Q0640m
pseudodementia of depression, the patient often seems concerned or even frustrated about the
• 33

Exit
• 7 a Item: 36 of 44 <J C> il , . ~
• 8 Prev1ous Next Lab Values Notes Calculator

• 9
0 E. The patient tries hard to answer questions about orientation and registration
• 1.0
• 1.1.
• :12
• 1.3
Explanation ReKaps & Refs 0 Help Q My Questions

• 1.4
The correct answer is B. Distinguishing a pseudodementia of depression from a true 69% of students got this correct
• 1.5
dementia is an important task requiring examination of subtle aspects of the mental status
• 1.6 Send us your feedback at:
examination because treatment for the two conditions would be very different, and memory loss medFeedback@kaplan.com
• 1.7
in this woman should not be automatically presumed to be related to a primary dementia. In
• 1.8
pseudodementia of depression, the patient often seems concerned or even frustrated about the Please include QID Q0640m
• 1.9 memory loss. whereas patients with true dementia tend to hide their memory-loss problems.
• 20
• 21. If the patient appears unconcerned (choice A) about the memory loss. it would be more
characteristic of dementia. Usually in pseudodementia of depression, the patient communicates
• 22
a strong sense of distress.
• 23
• 24 If the patient has more difficully with recent memory than remote memory (choice C) , true
• 25 dementia should be considered. In pseudodementia of depression, there is usually significant
• 26 memory loss for both recent and remote events.
• 27
If the patient tends to conceal the difficully (choice D) . it would be more characteristic of
• 28 dementia.
• 29
If the patient tries hard to answer questions about orientation and registration (choice E) •
• 30
dementia should be considered, since pseudodementia of depression is often accompanied by
• 31.
little effort to perform even simple tasks.
• 32
• 33

Exit
• 8
• 9
• 10 A 45-year-old woman comes to her primary care physician because of blurred vision. She states that this symptom started about 2
• 11 days ago. She denies any past history of significant medical or neurologic problems. She does state that several days ago she started
• 12 treatment for depression with a psychiatrist, due to a 2-month period during which she had several depressive symptoms. On review
• 13 of systems, she admits to having increased her water consumption over the last several days due to a dry mouth. She also complains
• 14 of dizziness when she stands up from lying or sitting. Her temperature is 37• C (98.6° F). blood pressure lying down is 135/75 mm Hg,
• 15 blood pressure standing is 110/64 mm Hg, pulse lying down is 84/min, pulse standing is 95/min, and respiratory rate is 16/min.
• 16 Physical examination is unremarkable except for mild pupillary dilation. Which of the following medications most likely accounts for

• 17 this patienrs symptoms?

• 18
• 19
0 A. Bupropion
• 20
• 21 0 B. Citalopram
• 22 (I 0 C. lmtpramme
• 23
• 24 0 D. Nefazodone

• 25 0 E. Sertraline
• 26
• 27
• 28 Explanation Rel<aps & Refs @ Help 0 My Questions
• 29
• 30 The correct answer is C. Imipramine is a tricyclic antidepressant (TCA) that inhibits 38% of students got this correct.
• 31 norepinephrine and serotonin reuptake. Like most TCAs, imipramine also has antagonistic
32 Send us your feedback at:
• effects at muscarinic , histaminic, and a-adrenergic receptors. This patient's compla int of blurred
medFeedback@kaplan.com
• 33 vision is most likely due to the medication's antimuscarinic effects. The blockade of muscarinic
Please includr ..... ,...,.. ......... _
"~
• 34
• 35
• 36
Exit
• '¥/
• 8
• 9
• 10 0
Explanation ReKaps & Refs ~ Help My Questions
• 11
• 12
The correct answer is C. Imipramine is a tricyclic antidepressant (TCA) that inhibits 38% of students got th is correct.
• 13 norepinephrine and serotonin reuptake. Like most TCAs, imipramine also has antagonistic
• 14 effects at muscarinic, histaminic, and a-adrenergic receptors. This patient's complaint of blurred
Send us your feedback a t:
medFeedback@kaplan.com
• 15 vision is most likely due to the medication's antimuscarinic effects. The blockade of muscarinic
• 16 Please include QID Q0455m
acetylcholine receptors causes mydriasis (pupillary dilation), resulting in blurred vision. Dry
• 17 mouth is also due to the anticholinergic effect of imipramine. Orthostatic hypotension is caused
• 18 by the a 1 -adrenergic receptor blockade associated with TCAs.
• 19
Bupropion (choice A) is an antidepressant and is used as an aide for smoking cessation.
• 20
Bupropion side effects do not include orthostatic hypotension or anticholinergic side effects.
• 21
Mechanism of action is unknown, though some evidence suggests that it is a norepinephrine
• 22
and dopamine reuptake inhibitor.
• 23
• 24 Citalopram (choice B) is a selective serotonin reuptake inhibitor (SSRI) with no significant
• 25 effects at cholinergic, adrenergic, or histaminic receptors. Therefore, it would not cause the
• 26 cluster of symptoms seen in this patient
• 27 Nefazodone (choice D) acts primarily as an antagonist at serotonin-2 (5-HT ) receptors,
2
• 28 although it is also a weak inhibitor of serotonin reuptake. Although it is related to trazodone, it
• 29 lacks significant antagonistic activity at other receptors.
• 30
Sertraline (choice E) is an SSRI, like citalopram, and would not cause the side effects present
• 31
in this patient
• 32
• 33 Reviewed on 1216112
• 34
• 35
• 36
Exit
• '¥/
• 9
• 10
• 11 A 26-year-old single, unemployed Caucasian man who has a history of paranoid schizophrenia presents to the outpatient clinic . He
• 12 reports that his previous psychiatrist prescribed risperidone, 2 mg/d, but he does not take it regularly. He states that it makes him tired
• 13 and occasionally stiff, and he does not think he needs to take medication on a daily basis. He is mildly disorganized and somewhat
• 14 disheveled but denies acute psychotic symptoms or thoughts of self-harm. Which of the following interventions should be tried next?
• 15
• 16
• 17 (I 0 A. Depot medication
• 18 0 B. Hospitalization
• 19
0 C. Increase risperidone to 4mg/d
• 20
• 21 0 D. Mental health court hearing
• 22
0 E. Switch to aripiprazole
• 23
• 24
• 25
Explanation ReKaps & Refs f) Help Q My Questions
• 26
• 27
The correct answer is A. This patient who has schizophrenia presents as noncompliant with 50% of students got this correct.
• 28
oral antipsychotic medications. There are a number of long-acting, depot neuroleptic
• 29 Send us your feedback at:
medications, including haloperadol, fl uphenazine and risperidone that are administered medFeedback@kaplan.com
• 30 intramuscularly every 2 weeks or monthly . These depot preparations have been shown to be
• 31 efficacious and helpful in ensuring medication compliance in individuals who have
Please include QJO m001059

• 32 schizophrenia and who are noncompliant with oral medication .


• 33
Hospitalization (choice B) is not necessary as the next step in this case, because the patient
• 34
does not have acute psychotic symptoms or suicidal ideation that would require this level of care
• 35
• 36
Exit
• 9
• 10
• 11 0 E. Switch to aripiprazole
• 12
• 13
• 14 Explanation ReKaps & Refs t} Help Q My Questions
• 15 '

• 16 The correct answer is A. This patient who has schizophrenia presents as noncompliant with 50% of students got this correct
• 17 oral antipsychotic medications. There are a number of long-acting, depot neuroleptic
Send us your feedback at:
• 18 medications, including haloperadol, fluphenazine and risperidone that are administered medFeedback@kaplan.com
• 19 intramuscularly every 2 weeks or monthly. These depot preparations have been shown to be
Please include QID m001059
• 20 efficacious and helpful in ensuring medication compliance in indMduals who have
• 21 schizophrenia and who are noncompliant with oral medication.
• 22 Hospitalization (choice B) is not necessary as the next step in this case, because the patient
• 23 does not have acute psychotic symptoms or suicidal ideation that would require this level of care
• 24 at this time.
• 25
Increasing the risperidone dosage (choice C) is not appropriate, because he is complaining
• 26
of side effects and is not compliant with oral risperidone at this time.
• 27
• 28 Mental hea~h court hearings (choice D) are not always available in the community but might
• 29 be tried if this patient is noncompliant with depot medication.
• 30
Switching to aripiprazole (choice E) is not correct, because this young man is not compliant
• 31 with oral medication at this time. Depot medication is a better next step at this point in his
• 32 treatment
• 33

Exit
• 10
• 11
• 12 A 32-year-old man is brought to the emergency department by a friend because he has been euphoric, sedated, and indifferent to
• 13 pain since last night. Physical examination is significant for a constricted pupils. This is the third time that the patient has been brought
• 14 to this hospital for this reason. After the administration of appropriate treatment and a complete physical examination, which of the
• 15 following laboratory studies is most important in the evaluation of this patient to provide long-term follow-up care?
• 16
• 17
• 18 0 A. Albumin level
• 19 0 B. Echocardiogram
• 20
• 21 0 C. Electroencephalogram

• 22 fl 0 D. HIV antibody test


• 23
0 E. Plasma liver enzyme levels
• 24
• 25
• 26
• 27
Explanation Rel<aps & Refs @ Help 0 My Questions

• 28
The correct answer is D. This patient is a heroin abuser, diagnosed from the signs and 70% of students got this correct.
• 29
symptoms of euphoria, sedation, indifference to pain, and pupillary constriction. Because of the
• 30 Send us your feedback at:
frequent use of needles in a patient with heroin dependence, it is absolutely necessary to rule medFeedback@kaplan.com
• 31
out the possibility of HIV infection. This is especially true with a patient who is unable to provide
• 32 a history during an episode of heroin overdose. Concom itant HIV infection and the need for Please include QID Q040 lm
• 33 diagnosis will make this laboratory study the most important of all the studies listed.
• 34
35 Albumin level (choice A) and other plasma liver enzyme levels (choice E) are not usually

affected by an episode of heroin intoxication.
• 36
• 37
• 38
Exit
• 39
• 10
• 11
• 12
0 C. Electroencephalogram
• 13
• 14 fl 0 D. HIV antibody test
• 15
0 E. Plasma liver enzyme levels
• 16
• 17
• 18
• 19
Explanation ReKaps & Refs 6 Help 0 My Questions

• 20
The correct answer is D. This patient is a heroin abuser, diagnosed from the signs and 70% of students got this correct.
• 21
symptoms of euphoria, sedation, indifference to pain, and pupillary constriction. Because of the
• 22 Send us your feedback at:
frequent use of needles in a patient with heroin dependence, it is absolutely necessary to rule medFeedback@kaplan.com
• 23 out the possibility of HIV infection. This is especially true with a patient who is unable to provide
• 24 a history during an episode of heroin overdose. Concomitant HIV infection and the need for Please include QID Q040 1m

• 25 diagnosis will make this laboratory study the most important of all the studies listed.
• 26
Albumin level (choice A} and other plasma liver enzyme levels (choice E) are not usually
• 27
affected by an episode of heroin intoxication.
• 28
• 29 An echocardiogram (choice B) may be indicated in a chronic IV drug abuser if there is
• 30 evidence of a murmur on auscultation of the heart, pointing to the possibility of bacterial
• 31 endocarditis.
• 32 An electroencephalogram (choice C) is not necessarily indicated in a patient with heroin
• 33 intoxication.
• 34
• 35
• 36
• 37
• 38
Exit
• 39
• 11
• 12
• 13 A 19-year-old obese girl comes to her physician because she often feels tired and sleeps throughout the course of the day. She says
• 14 that she sleeps for about 7 hours each night, and snores while sleeping. A few weeks ago, she collapsed to the floor when she met her
• 15 father for the first time after he had a stroke. She has no significant medical history and denies substance use. Her vital signs are
• 16 within normal limits. Her physical examination appears normal except for obesity. Which of the following is the most appropriate
• 17 treatment for this condition?
• 18
• 19
• 20 0 A. Benzodiazepine

• 21 0 B. CPAP
• 22
0 C. Ethosuccimide
• 23
• 24 0 0 D. Modafin11
• 25
0 E. No treatment is required at this time
• 26
• 27
• 28
• 29
Explanation Rel<aps & Refs 0 Help 0 My Questions

• 30
The correct answer is D. The diagnosis is narcolepsy, as cataplexy is described in the stem. 84% of students got this correct.
• 31
Cataplexy is pathognomonic for narcolepsy and consists of a sudden loss of muscle tone
• 32 precipitated by a loud noise or intense emotion (in this case, seeing her father for the first time
Send us your feedback at:
medFeedback@kaplan.com
• 33 after his stroke). Narcolepsy is a sleep disturbance characterized by excessive daytime
• 34 sleepiness and abnormalities of REM sleep. Other symptoms of narcolepsy include sleep
Please include QIO Q0400m

• 35 attacks, hypnagogic and hypnopompic hallucinations. and sleep paralysis.


• 36
The treatment of choice for narcolepsy is modafinil, a non-stimulating stimulant which will not


Exit
• 11
• 12
• 13
• 14
• 15
• 16
Explanation ReKaps & Refs 6 Help 0 My Questions

• 17
The correct answer is D. The diagnosis is narcolepsy, as cataplexy is described in the stem. 84')(, of students got this correct.
• 18
Cataplexy is pathognomonic for narcolepsy and consists of a sudden loss of muscle tone
• 19 Send us your feedback at:
precipitated by a loud noise or intense emotion (in this case, seeing her father for the first time
• 20 medFeedback@kaplan.com
after his stroke). Narcolepsy is a sleep disturbance characterized by excessive daytime
• 21 Please include QID Q0400m
sleepiness and abnormalities of REM sleep. Other symptoms of narcolepsy include sleep
• 22 attacks, hypnagogic and hypnopompic hallucinations, and sleep paralysis.
• 23
• 24 The treatment of choice for narcolepsy is modafinil, a non-stimulating stimulant which will not

• 25 increase the risk of addiction.

• 26 Benzodiazepine (choice A) is used in sleep disorders such as insomnia, night terror, and
• 27 sleepwalking. It has no role in narcolepsy.
• 28
CPAP (choice B ) is the treatment for sleep apnea Although this patient could exhibit
• 29
obstructive sleep apnea through appropriate sleep studies, cataplexy is not a feature of sleep
• 30
apnea.
• 31
• 32 Ethosuccimide (choice C) is the drug of choice for a patient with absence seizures. In
• 33 absence seizures the person would be staring into space with or without jerking movements of

• 34 the eye ball.

• 35 Doing nothing (choice E) is not the right answer; narcolepsy should be treated .
• 36
Reviewed on 12/10112

Exit
• 12
• 13
• 14 A 30-year-old single woman who works as a computer programmer for a large corporation presents for outpatient treatment. She
• 15 describes a long-standing history of feelings of low self-esteem, difficulty form ing satisfying relationships, and inadequacy in her
• 16 work. She tries not to attend work-related meetings and dreads interactions with coworkers, preferring to stay in her office. She feels
• 17 that her coworkers think she isn't as qualified as other programmers and is very sensitive to criticism by her peers. She sometimes
• 18 thinks that coworkers speak about her behind her back. She would like to have more friends, both at the office and in her community,
• 19 but tends to feel inferior to others. This presentation is most consistent with a diagnosis of which of the following?
• 20
• 21
• 22 (I 0 A. Avoidant personality disorder

• 23 0 B. Dependent personality disorder


• 24
0 C. Paranoid personality disorder
• 25
• 26 0 D. Schizoid personality disorder
• 27
0 E. Schizotypal personality disorder
• 28
• 29
• 30
• 31
Explanation Rel<aps & Refs G Help Q My Questions

• 32
The correct answer is A. This young woman presents with a long history of avoiding activities 67% of students got this correct
• 33
that involVe significant interpersonal contact because of fea rs of criticism or disapproval. She
• 34 Send us your feedback at:
would like more interactions in social and occupational settings but is held back and inhibited.
medFeedback@kaplan .com
• 35 This pattern of behavior is consistent with a diagnosis of avoidant personality disorder.
• 36 Please include QID m001060

• 37 Individuals who have dependent personality disorder (choice B) have a pervasive need to be
taken care of that leads to submissive and clinging behaviors.
• 38
• 39
• 40
Exit
• 4 1.
• 12
• 13
• 14
• 15 0 C. Paranoid personality disorder
• 16
0 D. Schizoid personality disorder
• 17
• 18 0 E. Schizotypal personality disorder
• 19
• 20
• 21 Explanation ReKaps & Refs @ Help 0 My Questions
• 22
• 23 The correct answer is A. This young woman presents with a long history of avoiding actMties 67% of students got tflis correct.

• 24 that involve significant interpersonal contact because of fears of criticism or disapproval. She
Send us y our feedback at:
• 25 would like more interactions in social and occupational settings but is held back and inhibited.
medFeedback@kaplan.com
• 26 This pattern of behavior is consistent with a diagnosis of avoidant personality disorder.
Please include QID m001060
• 27 Individuals who have dependent personality disorder (choice B) have a pervasive need to be
• 28 taken care of that leads to submissive and clinging behaviors .
• 29
Individuals who have paranoid personality disorder (choice C) have a pervasive distrust of
• 30
others that leads them to be suspicious, bear grudges, and be reluctant to confide in others.
• 31
• 32 Individuals who have schizoid personality disorder (choice D) do not desire close
• 33 relationships and appear indifferent to the c ritic ism of others.
• 34 Individuals who have schizotypal personality disorder (choice E) have odd beliefs, magical
• 35 thinking, and unusual perceptual experiences in addition to a lack of close friends.
• 36
• 37
• 38
• 39
• 40
Exit
• 4 1.
• 13
• 14
• 15 A 42-year-old married man presents to the outpatient clinic. He reports that he was diagnosed in his late teens with schizophrenia,
• 16 paranoid type, after suffering from persecutory delusions, auditory hallucinations, and poor school and occupational functioning. He
• 17 reports that he has a number of stressors at this time, including the death of his father and problems with his wife. He has a family
• 18 psychiatric history significant for major depressive disorder on his mother's side of the family. He is currently stable on aripiprazole,
• 19 15 mg/d, and he works in a factory. Which of the following features of this patient is associated with a poor prognosis?
• 20
• 21
• 22 0 A. Being married

• 23 0 B. Family history of a mood disorder


• 24
0 C. History of positive symptoms
• 25
• 26 0 0 D. Onset of symptoms at an ear1y age
• 27
0 E. Precipitating stressors at the start of psychotic symptoms
• 28
• 29 0 F. Quick onset of symptoms
• 30
• 31
• 32 Explanation Rel<aps & Refs 8 Help 0 My Questions
• 33
• 34 The correct answer is D. Schizophrenia is a psychiatric illness characterized by symptoms of 86% of students got this correct.

• 35 delusions, auditory hallucinations, and disorganization that last for more than 6 months and
Send us your feedback at:
• 36 result in significant social and occupational dysfunction. Studies on the course of the illness medFeedback@kaplan.com
• 37 have found several factors that are associated with better or worse prognosis. The following are
Please include QIO m001046
lists of factors associated with a better or worse prognosis.
• 38
• 39 n-"-- n------:-

Exit
• 13
• 14
• 15 The correct answer is D. Schizophrenia is a psychiatric illness characterized by symptoms of 86% of students got this correct.
• 16 delusions, auditory hallucinations, and disorganization that last for more than 6 months and
Send us your feedback a t:
• 17 result in significant social and occupational dysfunction. Studies on the course of the illness medFeedback@kaplan.com
• 18 have found several factors that are associated with better or worse prognosis. The following are
Please include QID m001046
• 19 lists of factors associated with a better or worse prognosis.
• 20 Better Prognosis
• 21 Good premorbid adjustment
• 22 Female
• 23 Married
• 24 Later age of onset
• 25 Precipitating events
• 26 Positive symptoms
• 27 Family history of mood disorder
• 28 Good inter-episode functioning
• 29 Quick onset of symptoms
• 30 Worse Prognosis
• 31 Poor premorbid adjustment
• 32 Male
• 33 Single
• 34 Earlier age of onset
• 35 Negative symptoms
• 36 Family history of schizophrenia
• 37 Poor inter-episode functioning
• 38 Family history of a mood disorder (choice B) is not a factor in the prognosis of schizophrenia,
• 39

Exit
• 13
• 14
• 15 Female
• 16 Married
• 17 Later age of onset
• 18 Precipitating events
• 19 Positive symptoms
• 20 Family history of mood disorder
• 21 Good inter-episode functioning
• 22 Quick onset of symptoms

• 23 Worse Prognosis
• 24 Poor premorbid adjustment
• 25 Male
• 26 Single
• 27 Ear1ier age of onset
• 28 Negative symptoms
• 29 Family history of schizophrenia
• 30 Poor inter-episode functioning
• 31 Family history of a mood disorder {choice B) is not a factor in the prognosis of schizophrenia,
• 32 but family history of schizophrenia is .
• 33
Being married (choice A) , history of positive symptoms (choice C) , precipitating stressors at
• 34
the start of psychotic symptoms {choice E) , and quick onset (choice F) are all factors that
• 35
suggest a good prognosis. Only early age of onset (late teens) for this patient suggests a poor
• 36
outcome.
• 37
• 38
• 39

Exit
• 14
• 15
• 16 A 35-year-old woman is brought to the emergency department because she is stiff and unable to swallow or talk. She is confused and
• 17 tremulous, and her family is concerned because she developed a high fever last night and they suspect she has developed an
• 18 infection. Her past medical history is significant for bipolar disorder for 10 years. She was recently treated in the hospital for an acute

• 19 manic episode and started on lithium. She had an elective cholecystectomy 1 week ago. In the past couple of days, her family reports
that she has appeared "strange." Her temperature is 39.5 •c (103.1 °F), blood pressure is 160/108 mm Hg, pulse is 124/min, and
• 20
respirations are 24/min. She appears tremulous, diaphoretic, and confused. She has increased tone in her neck and extremities. Her
• 21
leukocyte count is 21 ,200/mm3 and the serum creatinine phosphokinase is 5,238 U/L. A thorough evaluation for infection is negative.
• 22
Which of the following should be the management?
• 23
• 24
• 25
0 A. Antibiotics for meningitis

• 26 f) 0 B. Dantrolene
• 27
0 C. Dialysis
• 28
• 29 0 D. Halothane
• 30 0 E. Succinylcholine
• 31
• 32
• 33
Explanation Rel<aps & Refs G, Help Q My Questions
• 34
• 35 The correct answer is B . This patient's clinical presentation is suggestive of neuroleptic 88% of students got this correct
• 36 malignant syndrome (NMS), which is typically characterized by an altered mental status, fever,
• 37 tremor, dysautonomy (elevated or labile blood pressure, diaphoresis, tachycardia), and rigidity.
Send us your feedback at:
medFeedback@kaplan.com
• 38 There is also an elevated creatinine phosphokinase (CPK). and there can be a leukocytosis as
• 39 well. NMS is triggered by dopamine blockade and is typically seen after administration of Please include QID s2s162m

• 40
• 41
• 42
Exit
• 43
• 14
• 15
• 16
• 17
The correct answer is B . This patient's clinical presentation is suggestive of neuroleptic 88% of students got this correct
• 18
malignant syndrome (NMS), which is typically characterized by an altered mental status, fever,
• 19 Send us your feedback at:
tremor, dysautonomy (elevated or labile blood pressure, diaphoresis, tachycardia), and rigidity.
• 20 medFeedback@kaplan.com
There is also an elevated creatinine phosphokinase (CPK}, and there can be a leukocytosis as
• 21 Please include QID s2sl62m
well. NMS is triggered by dopamine blockade and is typically seen after administration of
• 22
antipsychotic (neurolepti c) medications. This patient probably received haloperidol for the
• 23 treatment of acute manic episode in the hospital. Treatment is with cooling measures, hydration,
• 24 dantrolene, bromocriptine, and maintaining hemodynamic stability.
• 25
Antibiotics (choice A) would not help because even though meningitis can cause some of the
• 26
same symptoms, it would not be precipitated by administration of a neuroleptic medication.
• 27
Moreover, meningitis does not elevate the CPK.
• 28
• 29 Dialysis (choice C) is not appropriate because this is not a case of lithium toxicity. Lithium
• 30 toxicity does not present with autonomic instability, fever, high WBC count, and high CPK
• 31 levels.
• 32 Halothane (choice D) and succinylcholine (choice E) are not correct; this is not malignant
• 33 hyperthermia (MH), because the symptoms of MH start within hours of starting the trigger agent.
• 34 Here, the patient had a surgery a week ago Moreover, even if it were MH, treatment would not
• 35 be succinylcholine or halothane; the treatment would still be dantrolene. MH has several features
• 36 in common with NMS, such as muscle rigidity, hyperthermia , elevated creatine kinase, and
• 37 good response to dantrolene. MH, however, is induced by inhalant anesthetic and the
• 38 susceptibility is inherited.
• 39
• 40
• 41
• 42
Exit
• 43
• 15
• 16
• 17 A mother brings her 8-year-old son to a psychiatrist for new-onset enuresis. A prior workup to determine a medical cause was
• 18 negative. In conversation, it seems that the enuresis started following parental arguments and separation. The boy wets himself at least
• 19 twice a week and feels upset about it, refusing to go for a sleep-over at his friend's house. Wh ich of the following is the most effective
• 20 first step in management for this condition?
• 21
• 22
• 23 (I 0 A. Behav~oral therapy
• 24 0 B. Interpersonal therapy
• 25
26 0 C.DDAVP

• 27 0 D. Imipramine
• 28
0 E. Psychotherapy
• 29
• 30
• 31
• 32
Explanation Rel<aps & Refs @ Help 0 My Questions

• 33
The correct answer is A. Behavioral therapy is the most frequently used treatment in children 15% of students got this correct.
• 34
with enuresis. Dry nights are recorded on a calendar and rewarded with a star as a gift. The
• 35 Send us your feedback at:
buzzer and pad apparatus are also used fo r conditioning nowadays but nonetheless, behavioral
medFeedback@kaplan.com
• 36
therapy should be tried first before starting medication.
• 37 Please include QIO Q0474m
• 38 Interpersonal therapy (choice B) is short-term therapy developed for nonpsychotic, milder
• 39 forms of depression. It addresses current relationships and roles, and is used with adults. It is
• 40 not indicated for enuresis treatment.

• 41

Exit
• 15
• 16
• 17
• 18 0 E. Psychotherapy
• 19
• 20
• 21 Explanati on ReKaps & Refs G Help Q My Questions
• 22
• 23 The c orrect answer is A. Behavioral therapy is the most frequently used treatment in children 15% of students got this correcL
• 24 with enuresis. Dry nights are recorded on a calendar and rewarded with a star as a gift. The
Send us your feedback at:
• 25 buzzer and pad apparatus are also used for conditioning nowadays but nonetheless. behavioral medFeedback@kaplan.com
• 26 therapy should be tried first before starting medication.
Please include QID Q0474m
• 27 Interpersonal therapy (choice B) is short-term therapy developed for nonpsychotic, milder
• 28 forms of depression. It addresses current relationships and roles , and is used with adults. It is
• 29 not indicated for enuresis treatment
• 30
DDAVP (choice C) is the preferred medication used when behavioral therapy fails . One
• 31
serious adverse effect of desmopressin is seizure or other CNS symptoms due to water
• 32
intoxication.
• 33
• 34 Imipramine (choice D) is another medication used when behavioral therapy fails. Adverse
• 35 effects include constipation, difficulty in initiating vo iding, irritability, drowsiness, reduced
• 36 appetite, and personality changes.
• 37 Psychotherapy (choice E) is not recommended unless there is evidence of other
• 38 psychopathology. The exploration of conflicts in enuresis has shown little success.
• 39
• 40
• 41

Exit

You might also like